0% found this document useful (0 votes)
95 views

Recall A

1. A 50-year-old heavy alcoholic man presented with ascites, enlarged liver, abdominal tenderness, and fever. 2. The best initial treatment is paracentesis to obtain ascitic fluid for analysis and culture before starting antibiotics. 3. Given the patient profile, spontaneous bacterial peritonitis likely from gut bacteria such as E. coli is suspected, so a third-generation cephalosporin like cefotaxime would be an appropriate empiric antibiotic choice.

Uploaded by

Nikhil Walia
Copyright
© © All Rights Reserved
We take content rights seriously. If you suspect this is your content, claim it here.
Available Formats
Download as DOC, PDF, TXT or read online on Scribd
0% found this document useful (0 votes)
95 views

Recall A

1. A 50-year-old heavy alcoholic man presented with ascites, enlarged liver, abdominal tenderness, and fever. 2. The best initial treatment is paracentesis to obtain ascitic fluid for analysis and culture before starting antibiotics. 3. Given the patient profile, spontaneous bacterial peritonitis likely from gut bacteria such as E. coli is suspected, so a third-generation cephalosporin like cefotaxime would be an appropriate empiric antibiotic choice.

Uploaded by

Nikhil Walia
Copyright
© © All Rights Reserved
We take content rights seriously. If you suspect this is your content, claim it here.
Available Formats
Download as DOC, PDF, TXT or read online on Scribd
You are on page 1/ 86

1- Child 4yr with 2 day prodromal then harsh cough fever 39 nasal flaring, tracheal tug,intercostal

recession o2 sat 80% treatment

a)nebulized salbutamol,

b)nebulizedbedosinuoed

c),im adrenaline croup ??

d), iv benzyl penicillin

e)ivfluxaacillin

Fever will not be a distinguishing feature because fever (38-39 or even up to 40) is often seen in croup. for two
reasons I think croup fits the bill: (1) drooling is pretty common is epiglottitis which is not mentioned here and
(2) epidemiollogically epiglottitis is far less common in children after introduction of Hib vaccine. my other
argument is that although airway protection is the first priority but 

rushing to intubation without proper support is definitely wrong and is considered the most common pitfall
(ensure the availability of an anesthesiologist or other individual experienced in difficult intubation). on the
other hand oxygenation of patients with epiglottitis is of limited use because the airway is severely obstructed.
for me it is croup and adrenaline. If they wanted you to think of epilgolttitis they should have given you an extra
piece of information. DON'T JUMP ON CONCLUDING EPIGLOTTITIS JUST BECASUE OF FEVER. Fever is almost
always present in croup as well

There is no place for cough medicines or antibiotics. Steaming methods are discouraged

CROUP
JM P:1069
Differential diagnosis
The main alternative diagnosis is viral laryngo-tracheobronchitis (croup). There are, however,
significant clinical differences. Epiglottitis is characterised by fever, a soft voice, lack of a harsh cough, a
preference to sit quietly (rather than lie down) and especially by a soft stridor with a sonorous expiratory
component. Croup is distinguished by a harsh inspiratory stridor, a hoarse voice and brassy cough. Other
differential diagnoses include tonsillitis, infectious mononucleosis and bacterial tracheitis.
Grade 1 croup
Mild croup (barking cough, no stridor or stridor at rest without chest retraction, hoarse voice):
• manage at home if no signs of Grade 2 or 3 croup, keep child relaxed
• consider oral steroids (e.g. dexamethasone 0.15–0.3 mg/kg/dose or prednisolone 1 mg/kg/dose) if stridor and
chest wall retraction develop
Grade 2 croup
Moderate croup (inspiratory stridor when upset or at
rest with chest wall retractions):
• admit to hospital (e.g. emergency department)
• cool humidified air
• oral steroids dexamethasone 0.6 mg/kg or prednisolone (tablets or oral solution)1 mg/kg (2–3 doses)
and/or (for children 2 or more years)
budesonide 100 mcg  ×  20 puffs or 2 mg nebulised
• nebulised adrenaline—if poor response to steroids
• observe for at least 4 hours
Grade 3 croup
Severe croup (inspiratory stridor at rest, use of
accessory muscles, patient restless and agitated).
Adrenaline is first-line therapy:
• nurse in intensive care
• oxygen
• nebulised adrenaline 1:1000 solution 0.5 mL/kg/dose (to max. 5 mL) (beware of possible rebound effect after 2–
3 hours—must be observed)
Note: Can use 4 ampoules of 1:1000 solution in a
nebuliser run with oxygen 8 L/min. Repeat the dose if
no response at 10–15 minutes, do not dilute solution.
• dexamethasone 0.2 mg/kg IVor 0.6 mg/kg IM followed by oral steroids
• have facilities for artificial airway
• may need endotracheal intubation (if going into
respiratory failure) for 48 hours. Use a tube
0.5–1 mm smaller than normal for age.
There is no place for cough medicines or
antibiotics. Steaming methods are discouraged.
Barking cough with stridor=> croup (usually no fever, viral)

2-Recent coming from india, with bloody diarrhea and abd pain. On hx, he said he is suffering
with this for last 3 years. what's the cause?
a) E.Coli
b)Amoeba
c)Ulcerative colitis
d)Corhn colitis

medscape: Chronic amebic colitis is clinically similar to inflammatory bowel disease (IBD). Recurrent
episodes of bloody diarrhea and vague abdominal discomfort develop in 90% of patients with chronic
amebic colitis who have antibodies to E histolytica. Amebic colitis should be ruled out before treatment
of suspected IBD because corticosteroid therapy worsens amebiasis

Crohn disease
Synonyms: regional enteritis, granulomatous colitis. The cause is unknown but there is a genetic link.
Clinical features:
• Recurrent diarrhoea in a young person (15–40 years)
• Blood and mucus in stools (less than UC)
• Colicky abdominal pain (small bowel colic)
• Right iliac fossa pain (confused with appendicitis)
• Constitutional symptoms (e.g. fever, weight loss, malaise, anorexia, nausea)
• Signs include perianal disorders (e.g. anal fi ssure, fi stula, ischiorectal abscess), mouth ulcers
• Skip areas in bowel
Main symptom• Colicky abdominal pain
Diagnosis
• Sigmoidoscopy: ‘cobblestone’ appearance (patchy mucosal oedema)
• Colonoscopy: useful to differentiate from UC
• Biopsy with endoscopyPrognosisLess favourable than UC with both medical and surgical treatment.Management
principles
• Education and support including support groups
• Treat under consultant supervision
• Treatment of acute attacks depends on severity of the attack and the extent of the disorder:— mild attacks:
manage out of hospital— severe attacks: hospital, to attend to fluid and electrolyte balance
• Role of diet controversial: consider a high-fibre diet but maintain adequate nutrition
• Pharmaceutical agents (the following can be considered): — 5-aminosalicylic acid derivatives (mainly UC):
sulfasalazine (mainstay), olsalazine, mesalazine— corticosteroids (mainly for acute flares): oral, parenteral, topical
(rectal foam, suppositories or enemas)— immunomodifying drugs (e.g. azathioprine, cyclosporin, methotrexate)
and biological agents (e.g. infl iximab)
• Surgical treatment: reserve for complications
Ulcerative colitis
Clinical features
• Mainly a disease of Western societies
• Mainly in young adults (15–40 years)
• High-risk factors—family history, previous attacks, low-fi bre diet
• Recurrent attacks of loose stools
• Blood, or blood and pus, or mucus in stools
• Abdominal pain slight or absent
• Fever, malaise and weight loss uncommon
• Begins in rectum (continues proximally)—affects only the colon: it usually does not spread beyond the ileocaecal
valve
• An increased risk of carcinoma after 7–10 years
Main symptom• Bloody diarrhoea
Diagnosis
• Proctosigmoidoscopy: a granular red proctitis with contact bleeding
• Barium enema: characteristic changes
Prognosis
• 5% mortality in an acute attack
• Recurrent attacks common
(Jm p 470)

I’m more with CD

3-A 50 year old heavy alcoholic man came with ascites, enlarged liver 4 cm below costal
margin , his abdomen is tender and his temp. is 37.9 c his blood pressure is 100/60 which of the
following is best treatment for current problem
a. IV metronindazole + amoxicillin
b. IV metronindazole + amoxicillin + gentamycin (/)
c. Paracentesis
d. Oral amox. Clax.
e. IV albumin

TIP: peritonitis could present without abdominal tenderness in patients with ascites.
‫براى بيمار سيروتيك و اسيت بهترين و اولين قدم اينه كه پاراسنتز كنيم و بفرستيم واسه سايتولوژى و كشت و بعد انتى بيوتيك شروع كنيم‬.

‫ مراجعه کنن ولی پروتونئیت نداشته باشند‬A‫افرادی که الکلیک هستند و ثانویه به اون دچار سیروز و آسیت هستند ممکنه با درد شکم و تب و لکوسیتوز‬
.‫ دارن یا نه‬250 ‫ پلی مورف بیشتر از‬A‫رو این حساب حتما باید قبل آنتی بیوتیک تپ آسیت بشن که ببینم‬
‫ هایی که گذاشته ماله‬A‫ هرچند آنتی بیوتیک‬A.‫حاال اگه این سوال بیمار الکلیک نبود چون بهترین درمان رو میخواست جواب میشد شروع آتی بیوتیک‬
‫ خود بخودی درمان سفالوسپورین نسل سوم هست‬A‫ ثانویه هست و برای پریتونئت‬A‫پریتونئیت‬

INTRODUCTION — Spontaneous bacterial peritonitis (SBP) is defined as an ascitic fluid infection without an
evident intra-abdominal surgically treatable source; it occurs primarily in patients with advanced cirrhosis . The
diagnosis is established by a positive ascitic fluid bacterial culture and an elevated ascitic fluid absolute
polymorphonuclear leukocyte (PMN) count ≥250 cells/mm3. An important issue in differential diagnosis and
therapy is distinguishing SBP from a surgically treatable cause of secondary bacterial peritonitis (eg, due to a
perforated viscus).
Clinical setting — Patients with SBP generally have advanced cirrhosis. The higher the MELD score, the higher the
risk of SBP . Spontaneous infection of noncirrhotic ascites (eg, due to malignancy or heart failure) is unusual
enough to be the subject of case reports or small series.
Fever, abdominal pain, altered mental status,abdominal tendernes,diarrhea, paralytic ileus, hypotention,
hypothermia
Microbiology and choice of antibiotic — Most cases of SBP are due to gut bacteria such as Escherichia coli and
Klebsiella; however, streptococcal and, infrequently, staphylococcal infections can also occur . As a result,
relatively broad-spectrum therapy is warranted in patients with suspected ascitic fluid infection until the results of
susceptibility testing are available.
cefotaxime or a similar third-generation cephalosporin is a reasonable choice for suspected SBP
or the combination of ampicillin and gentamicin
TREATMENT — Empiric therapy of suspected SBP must be initiated as soon as possible to maximize the patient's
chance of survival.
The main indication for empiric therapy is the otherwise unexplained presence of one or more of the following
findings that are characteristically seen in SBP :

Temperature greater than 37.8ºC (100ºF)


Abdominal pain and/or tenderness
A change in mental status
Ascitic fluid PMN count ≥250 cells/mm3
Decision to treat — In patients with fever, abdominal pain or tenderness, or altered mental status, treatment can
be started as soon as ascitic fluid, blood, and urine have been obtained for culture and analysis. In patients without
these findings, it is reasonable to wait until the results of the PMN count are available . Collection and processing
of the specimen should take no more than one to four hours from the time of the paracentesis. Patients with
ascitic fluid PMN counts ≥250 cells/mm3 in a clinical setting compatible with ascitic fluid infection should receive
empiric antibiotic therapy.

The ascitic fluid PMN count is more rapidly available than the culture and appears to be a relatively reliable guide
as to who really needs empiric antibiotic coverage . Delaying treatment until the ascitic fluid culture grows bacteria
may result in death from overwhelming infection.

Patients with alcoholic hepatitis — Patients with alcoholic hepatitis regularly develop fever, leukocytosis, and
abdominal pain, which can mimic SBP. However, they also can develop SBP. Potentially contributing to the
diagnostic uncertainty is that patients with alcoholic hepatitis frequently have a peripheral leukocytosis.
Patients with a peripheral leukocytosis do not have a proportional increase in PMNs in ascites unless they also
have SBP [ 6 ]; thus an elevated ascitic fluid PMN count must be presumed to represent SBP. Empiric antibiotic
treatment (for presumed ascitic fluid infection) of patients with alcoholic hepatitis who have fever and/or
peripheral leukocytosis can be discontinued after 48 hours if ascitic fluid, blood, and urine cultures demonstrate
no bacterial growth.

Start ab therapy even if the pt is alcoholic do paracentesis and if the tap fluid is neg for pmn(<250) cease ab

4-69 yrs female with vaginal discharge noticed on her panty. PCR shows gonorrhea , pt. said she
didn’t have any sexual activity last 10 years , what is the best next ?
A) repeat PCR
B) give ceftriaxone single dose
C) Give azithromycine single dose
D) DO hysteroscopy
e)counsel about treatment
5-an old man was found unconscious, after regaining consciousness he said he is very cold,
temp 34.2 , BP 90/60, management?
Normal salin
Dopamine
Dextrose

On the other hand aside from probable hypoglycemia it could be due to SIRS or … hypotension become
more prominent and therapy would be N.S

‫ كه توى اورژانس براى كاهش سطح هوشيارى بايد‬TNG ‫ ولى كاهش سطح هوشيارى داشته مث همون ترتيب‬A‫وقتى فشار مريض خيلي پايين نيست‬
‫مدنظر داشته باشيم ميمونه‬
T thiamin
N naloxan
G glucose
٩٠/٦٠ ‫من برداشتم اينه كه احتمال هايپوگليسمى خطرناك تره تا فشار‬
‫ ولی از اونجایی‬.‫ البته قند رو هم باید حتما چک کنیم‬.‫ تو هایپوترمی اپروچ اینجوریه که باید اول نرمال سالین بدیم‬.‫درسته اما مسئله ما اینجا هیپوترمیه‬
‫ ولی اگه تو گزینه ها چک قند بود‬.‫که این سوال و همچنین گزینه هاش هیچ اشاره ای به قند نکردن باید همون اپروچ به هیپوترمی رو در نظر گرفت‬
.‫من مسلما اونو میزنم‬

HYPOTHERMIA
Core temp under 35 :
32-35 mild
28-32 moderate
<28 severe
Causes: Causes and factors contributing to the development of hypothermia include outdoor exposure, cold water
submersion, medical conditions (eg, hypothyroidism, sepsis), toxins (eg, ethanol abuse), and medications (eg, oral
antihyperglycemics, sedative-hypnotics). Risk increases if the ability to autoregulate core temperature is impaired
by medications or underlying disease, as is common among elderly patients.
Treatment: The initial management of hypothermia is directed toward resuscitation, rewarming. Endotracheal
intubation is performed in patients with respiratory distress or who cannot protect their airway. Patients with
moderate or severe hypothermia frequently become hypotensive; aggressive fluid resuscitation is appropriate
 Passive external rewarming is the treatment of choice for mild hypothermia.(remove wet clothes and
cover the pt in blanket, room temp:24)
 Active external rewarming is used to treat patients with moderate hypothermia, refractory mild
hypothermia, and as an adjunct in severe hypothermia. It consists of some combination of warm blankets,
radiant heat, or forced warm air applied directly to the patient's skin. Rewarming of the trunk should be
undertaken BEFORE the extremities to minimize hypotension and acidemia due to arterial vasodilation
and core temperature afterdrop.
 For severe and recalcitrant moderate hypothermia, we suggest initiating treatment with less invasive
rewarming techniques (eg, warmed IV crystalloid), and progressively adding more invasive ones (eg,
warmed pleural lavage) as needed

6-A lady had twin pregnancy. After delivery of the first baby which one of the following is the
next best step to do?
A-ARM of the second sac
B-PVE to chk the position of the second fetus
C-Wait for spontaneous delivery of the second baby

Second twin — After delivery of the first twin, the heart rate and position of the second twin should be evaluated
using ultrasound and electronic fetal monitoring. If the second twin is in a nonvertex presentation, ultrasound can
be used to assist external cephalic version, breech extraction, or internal podalic version of the second twin, if
necessary.
Interval between delivery of the two twins — Historically, second born twins were reported to have a higher
incidence of adverse outcome (morbidity and mortality) due to lower birth weight; higher frequency of
malpresentation, cord prolapse, and abruptio placentae; and more deliveries involving internal podalic version. A
prolonged interval between delivery of the first and second twins was also thought to be associated with poorer
outcomes. Intervals of less than 25 to 30 minutes were advocated, and maneuvers such as internal podalic version
or breech extraction of the second twin were often recommended to hasten delivery

7-There was an x-ray of bamboo spine with a patient with history of low back pain. Patient was
on NSAID. Asked for next best step
A-Hydrotherapy
B-Physiotherapy
C-MTX
D-Prednisolone

Treatment
indomethacin in optimal dosage
• sulphasalazine—a useful second-line agent if thedisease progresses despite NSAIDs
• DMARDs (e.g. methotrexate) or bDMARDs may benecessary in severe progressive disease

•in cases of ankylosing spondylitis with positive HLA-B27 the risk to offspring is signifi cant
• Acute anterior uveitis requires prompt treatment and monitoring by an ophthalmologist.
• Refer for physiotherapy for exercises, stretching program, postural exercises and hydrotherapy. Appropriate
physiotherapy slows deterioration in spinal function.
• Pharmacological agents:— NSAIDs (e.g. indomethacin 75–200 mg (o) or 100 mg rectally nocte daily or ketoprofen
100 mg rectally nocte to control pain, stiffness and synovitis)
— sulphasalazine (if NSAIDs ineffective)
— intra-articular corticosteroids for severe monoarthritis and intralesional corticosteroids for enthesopathyRefer
for advice on above and especially for DMARD and bDMARD therapy. (JM P 348 5th ed)
‫ چون فيزيوتراپى و درمانهاى كمكى واسه قبل از بامبو شدنه‬,mtx ‫اگه ايمفليكسماب& بود اون ميشه وگرنه‬
. ‫ اثر دارن‬peripheral synovitis ‫گايدالين هم ميگه قدم بعد اينفليكسيمب& هست و متوتركسات& و سولفاساالزين بيشتر رو‬

ANKYLOSING SPONDYLITIS
JM P:374 6th ed
This usually presents with an insidious onset of inflammatory back and buttock pain (sacroiliac joints and
spine) and stiffness in young adults (age <40 years), and 20% present with peripheral joint involvement before
the onset of back pain. It usually affects the girdle joints (hips and shoulders), knees or ankles. At some stage over
35% have joints other than the spine affected. The symptoms are responsive to NSAIDs.
Key clinical criteria
• Low back pain persisting for >3 months
• Associated morning stiffness >30 minutes
• Awoken with pain during second half of night
• Improvement with exercise and not relieved by rest
• Limitation of lumbar spine motion in sagittal and frontal planes
• Chest expansion ↓ relative to normal values
• Unilateral sacroiliitis (grade 3 to 4)
• Bilateral sacroiliitis (grade 2 to 4)
Treatment uptodate:
•We recommend use of an NSAID as initial therapy.
•We recommend an exercise program for all patients.
•We suggest NOT using systemic glucocorticoids. We suggest intraarticular glucocorticoids for persistent
peripheral joint involvement, enthesitis at sites other than the Achilles tendon, and for pain of sacroiliitis.
•Traditional nonbiologic DMARDs (eg, sulfasalazine , methotrexate , leflunomide , or penicillamine ) are ineffective
for those with axial disease treat with anti-TNF agent.
•For patients with axial disease who do not respond to NSAIDs we recommend an anti-TNF agent.
•In patients with predominantly peripheral arthritis who do not respond adequately to NSAIDs and for whom a
TNF inhibitor is unavailable or is contraindicated, we recommend sulfasalazine (alternatively methotrexate),
unless there are contraindications (eg, allergy to sulfonamide antibiotics) to this drug.
8-Child with rectal prolapse ' GI symptoms ( nausea ' vomiting ' abdominal ) ' btw and Height 50
percentile !!
A. H/o of sister with hook worm infection
B. Sweat chloride test
C. Duodenal biopsy
D. Anti glaitin A
‫ چون اگه‬.‫ ساله و عالیم ریوی هم نگفته باشه بزنیم سلیاک‬1 ‫ اگه گفته& باشه بچه باالی‬CF‫ باید عالیم ریوی‬،‫ سال باشه‬1 ‫ باشه و سن باالی‬CF ‫رو هم داشته باشه‬

9-contraception in mental disorders :


IM depo provera, if not in choices , implanon (Etonogestrel Subdermal: Implant 1 rod in the inner side of the
upper, nondominant arm. Remove no later than 3 years after the date of insertion. After ruling out pregnancy,
timing of insertion is based on the patient's contraceptive history)
NEVER USE IUD (MIRENA) ????
10- A 4 year child got admitted with fever, redenned and swollen tonsil, Antibiotic was started
but after few hours patient suddenly developed stridor. What is the next thing to do?
A-Intubation
B-Abscess drainage
C-Ivflucloxacillin

If pt developed stridor with an AB it might be an alarming symptom of angioedema and


impending airway obstruction thus the priority sequence would be serum therapy plus
adrenalin => intubation

11- . A patient with diabetes, hypertension, AF on perindropil, gliclazide, metformin, warferin came with
a red, hot tender swelling in his thigh. What will you do next ?
A-Stop warferin
B-Give iv antibiotic‫؟؟‬
C-Enoxaparin
D-Fresh plasma

:‫بچه ها این باالخره هماتوم شد یا سلولیت‬


‫اگر مریض امیودارون هم مصرف میکرد چون با وارفارین شانس خونریزی رو میبره باال میشه هماتوم ولی اینجا نگفته امیودارون پس احتمال‬
‫سلولیت باالس‬

12-35. A mother anxious as there is meningitis outbreak, brought her 5 year old son,
rhinorrhea, mild fever, on examination, red tonsil with pink tympanic membrane, no neck
stiffness. What would be most appropriate?
a. CT head b. LP c. Oral amoxicillin??
d. Quickly do nasopharyngeal aspirate to confirm viral cause

12.1-44-A mother came with her 18 months old baby who has 39 degree fever for the last 12 hours
reduced feeding but no skin rash. On examination he looks well unstressed or tired he has red tympanic
membranes with rednned tonsils .the mum concerned about out break of meningitis around. How to
manage :
1. Do nasopharyngeal swab for viral testing.
2. Give IV ceftriaxone
3 .do LP
4 . tell her to bring him back once he gets worse.
in JM page 290 .. he dosen’t have any signs of meningeal irritation (neck stiffness , headache , photophobia ,
vomiting ) moreover we start to suspect meningitis if fever > 3days in unstressed child thus presumably it’s an
otitis media. chemoprophylaxis for meningitis is discussed below but briefly its rifampin , ciprofloxacin , and
ceftriaxone (although using chemoprophylaxis have its own indications and criteria).since various viral infections
could mimic these scenario I think the next measure would be taking a nasopharyngeal swab and the best is
ceftriaxone (don’t forget the child is highly febrile and symptomatic for otitis, based on JM empirical therapy for
otitis media in this circumstance is required). Not to mention the best regimen here was amoxicillin prior to
cefriaxon.

CHEMOPROPHYLAXIS FOR MENINGITIS


Meningococcal meningitis
Close contacts — Chemoprophylaxis is indicated in close contacts of patients with meningococcal infection, and
should be given as early as possible following the exposure. Although "close contact" has not been clearly defined,
it generally refers to individuals who have had prolonged (>8 hours) contact while in close proximity (<3 ft) to the
patient, or who have been directly exposed to the patient's oral secretions between one week before the onset of
the patient's symptoms until 24 hours after initiation of appropriate antibiotic therapy [ 2 ].
Close contacts include:
 Household members, roommates, intimate contacts, individuals at a child-care center, young adults in
dormitories, military recruits in training centers, and sitting next to an index patient for more than eight
hours on an airplane.
 Individuals who have been exposed to oral secretions (eg, intimate kissing, mouth-to-mouth resuscitation,
endotracheal intubation or endotracheal tube management).
Prophylaxis is not indicated if exposure to the index case is brief. This includes the majority of healthcare workers
unless there is direct exposure to respiratory secretions (as with suctioning or intubation). The attack rate in
healthcare workers at risk is increased compared to the general population, but the absolute increase in risk is very
small and antimicrobial prophylaxis is not recommended
Timing: ideally <24 hours after identification of the index patient.Conversely, chemoprophylaxis
administered >14 days after exposure to the index case is probably of limited or no value
Regimens:
Haemophilus influenzae meningitis
 To prevent invasive disease in susceptible children, we recommend chemoprophylaxis for close contacts
of patients with invasive Hib disease who live in a household with a child who is younger than four years
of age and has not received an age-appropriate number of doses of Hib conjugate vaccine, or an
immunocompromised.
 We recommend chemoprophylaxis for child-care and preschool contacts when unimmunized or
incompletely immunized children attend the facility and two or more cases of Hib invasive disease have
occurred among attendees within 60 days.
 We recommend chemoprophylaxis for the index patient if he or she was treated with an agent other than
cefotaxime or ceftriaxone and is younger than two years of age or lives in a household with a child <4
years of age who has not received an age-appropriate number of doses of Hib conjugate vaccine, or an
immunocompromised child.
 We recommend rifampin for Hib chemoprophylaxis. The dose is 20 mg/kg (maximum dose 600 mg) once
per day for four days.
13-pregnant woman is going to visit shanghai next month, staying in a 5 star hotel. Proper
immunization?
HBV
Typhoid
influenza
Answer : C

14-Pregnant women is going to Asian country within 1m.which Vic is contatindicated?


A-Oral typhoid
B-Ipv
C-Hepatitis a
Contraindicated vaccines in pregnancy :
1.BCG
2.Oral typhoid
3.Japanese encephalitis
4.MMR
5.Rotavirus
6.Zoster

15- A farmer, returned from Thailand, still taking his antimalarial drugs, temp 37.4, lost weight 10 kg,
abdominal pain, dark color urine, asked diagnosis
a. Hydatid cyst
b. Pancreatic cancer
c. Liver abscess
d. Malaria
‫جهت صرفه جويى در وقت ‪ ،‬مطلب گايدالين رو مينويسم جهت پروفيالكسى& ماالريا ‪:‬‬
‫‪-atovaquone+proguanil‬‬
‫‪-doxycycline2days befor=>2w after 100mg/d‬‬
‫‪-mefloquine‬‬
‫‪chhloroquine 300mg/w 1w before=>4w after‬‬
‫‪ .‬البته بين اينها هيچ اولويتى قائل نشده كه كدوم اول كدوم دوم ولى گفته خيلى& جاها مثل تايلند& به مفلوكين مقاومت ايجاد شده و استفاده& نكنيد‬
‫‪ .‬هستن و قابل استفاده ‪ B‬هست در حاملگى& مابقى ‪ D‬بجز داكسى كه‬
16.1 schizophrenia patients wants to suicide an you are informed from her brother through land line?
Inform police
Inform legal assistance
Request talking to patient on the phone
If the person has a specific plan for suicide, or if they have the means to carry out their suicide plan, call a mental
health centre or crisis telephone line and ask for advice on the situation.If the suicidal person has a weapon,
contact the police. When contacting the police, inform them that the person is suicidal to help them respond
appropriately. Suicide Call Back Service
Support if you, or someone you know, is feeling suicidal.
Age: For all ages
Phone counselling 24/7

16.2 which has more neurological complications on fetus if consumed in late pregnancy?
TCA
Antipsychotics
Anticonvulsants
SSRI
antiepileptics
16.3 old woman with fall and rhabdomiolysis?
>1mg/kg
>2mg/kg
Normal urine output:
Adults 0.5cc/kg/hr
Child 1
Infant 2
But in rhabdomyolysis desired u/o is 200 ml/h in adults or 2cc/kg/hr (medscape)

16.4 hemorrhagic stroke with BP 180/100?


Intubation
Neurologic evaluation and observation
BP lowering
Manitol
In acute primary intracerebral haemorrhage where severe hypertension is observed on several occasions within
the first 24 to 48 hours of stroke onset, antihypertensive therapy (that could include intravenous treatment) can
be used to maintain a blood pressure below 180 mmHg systolic (mean arterial pressure of 130 mmHg).
There is currently insufficient evidence to recommend precise BP thresholds or targets in acute primary ICH. There
is a general consensus that severe sustained elevated BP (e.g. SBP>180 mmHg) can be treated, especially if there is
evidence or suspicion of raised intracranial pressure.

16.5 known case of DM, IHD (history of MI 2 years ago), on proper diet and exercise, labs normal but
chol:6.2 ?
Statin commencement
Continue the routine program
Normal cholesterol < 5.5
‫شروع استاتین ها‬
CAD :

Chol> 4 mmol
--------------------
FH Dm
FH CAD
HTN
PVD

Chol>6/5
Or
Chol > 5/5 & HDL< 1

16.6 diabetic patient with amputated foot, BP145/85, upper normal cholesterol, HB A1c ↑↑↑. What
do you suggest for prevention of amputation of the other foot?
Meticulous foot care
BP controlling
Glucose controlling
Lipid controling
Having had the side effect better to go for tight care of the other foot???

16.7 a child allergic to egg coming for MMR?


Do not
Do
Do under supervision
Contraindicated vaccines with egg allergy :
1-Influenza (some types are safe)
2-Q fever
3- PCECV type of Rabies vaccine (HDCV should be used instead)
4-Yellow fever
Children with egg allergy can be safely given MMR or MMRV vaccine.

16.8 patient on amiodaron, furesmide, aspirin comes with bradycardia.cause?


Amiodaron+foresmide
Amiodaron+aspirin
amiodarone and Lasix (furosemide) hypokalemia:
Talk to your doctor before using amiodarone together with furosemide. Combining these medications can increase
the risk of an irregular heart rhythm that may be serious. If your doctor prescribes these medications together, you
may need regular monitoring of your electrolyte (magnesium, potassium) levels as well as other tests to safely use
both medications. You should seek immediate medical attention if you develop sudden dizziness, lightheadedness,
fainting, or fast or pounding heartbeats during treatment with amiodarone. In addition, you should let your doctor
know if you experience signs of electrolyte disturbance such as weakness, tiredness, drowsiness, confusion, muscle
pain, cramps, dizziness, nausea, or vomiting. It is important to tell your doctor about all other medications you use,
including vitamins and herbs. Do not stop using any medications without first talking to your doctor.

16.9 pt with history of gastric banding comes with severe abd pain and dysphasia from 3 hours ago. Inv?
Abd CT
Abd Us
Barium swallow
Emergency surgery
The diagnosis is confirmed with an upper gastrointestinal series demonstrating no passage of contrast beyond the
band. If suspected to perforation use gastrografin.
‫عارضه جراحی معده که خودشو به صورت انساد نشون میده باریوم سوالو‬

16.10 a child with Trichotillomania difficult to cope with?


Refer to psychologist
A specific psychologic test…
the first line of treatment for trichotillomania is behavioral treatment and intervention. No medication has been
approved for the treatment of trichotillomania. Drug therapy has largely been disappointing, though some studies
have yielded encouraging results. A psychiatrist should be consulted when a serious psychiatric disorder is
suspected. Consultations with other specialists (eg, a psychologist or a developmental-behavioral pediatrics
specialist) should be obtained as indicated.

16.11 face trauma with inf orbital Fx most corroborating examination finding?
Decreased visual acquity
Opening mouth problem
Closing mouth problem
Numbness in the inferior orbital
Test the sensation over the lower lid skin. Loss of sensation indicates infra-orbital nerve injury, confirming a
blowout fracture.
trigeminal branch, infraorbital nerve
17-A girl after uni ex comes and says she couldn’t sleep in night before ex because of stress, so ex was
not good, now comes for certificate to reschedule that ex. Next?
a) Refer college medical practitioner
b) Contact ex coordinator
c) Cerficate only if she agrees to take antideppressan
d)refuse to give her certificate

18- 25 year old lady come at 8weeks gestation for her antenatal check up. Her mother has GDM and DM
now. which was diagnosed at 51 years age. What is the appropriate next step?
a. OGTT 20 weeks
b. OGTT 26-28weeks
c. OGCT 24-28 week
d. Fbs now

Moderate risk factor:


• Ethnicity: Asian, Indian subcontinent, Aboriginal, Torres Strait Islander, PacificIslander, Maori, Middle Eastern,
non‐white African
• BMI 25 – 35 kg/m2
Should initially bescreened with either a random or a fasting glucose test in early pregnancy, followed by a
pregnancy OGTT (POGTT) if clinically indicated.
High risk factors for GDM
• Previous GDM
• Previously elevated blood glucose level
• Maternal age ≥40 years
• Family history DM (1st degree relative with diabetes or a sister with GDM)
• BMI > 35 kg/m2
• Previous macrosomia (baby with birth weight > 4500 g or > 90th centile)
• Polycystic ovarian syndrome
• Medications: corticosteroids, antipsychotics
patients who have one high risk factor or two moderate risk factors
should undergo a 75 g POGTT, with venous plasma samples taken fasting, one hour
and two hours at the first opportunity after conception.
19- Patient with renal failure , uremia and increase creatinine, with PTh= 11 and phosphate very
increased ,calcium normal upper limit what to do?
a) Parathyroidectomy
b) phosphate restricted diet
c) ca supplementation
Dietary restriction alone may suffice for control of hyperphosphatemia in persons with mild renal insufficiency, but
it is inadequate for patients with advanced renal insufficiency or complete renal failure

20-A 4 y.o. girl presents with URTI . In urinalysis protein trace blood ++

IGA NEPHROPATHY
INTRODUCTION — IgA nephropathy is the most common lesion found to cause primary glomerulonephritis
throughout most developed countries of the world . Patients may present at any age, but there is a peak incidence
in the second and third decades of life. There is approximately a 2:1 male to female predominance in North
American and Western European populations, although this difference is not observed among populations in the
Pacific Rim.
The presence of IgA nephropathy is established only by kidney biopsy. The pathognomonic finding is observed on
immunofluorescence microscopy, which demonstrates prominent, globular deposits of IgA (often accompanied by
C3 and IgG) in the mesangium.
CLINICAL FEATURES — Patients with IgA nephropathy typically present in one of three ways; the relative frequency
depends in part upon screening practices (which will lead to increased discovery of asymptomatic cases) and the
particular population being evaluated :
1- Approximately 40 to 50 percent present with one or recurrent episodes of visible hematuria, usually following
an upper respiratory infection. This has sometimes been called 'synpharyngitic hematuria'. These episodes can be
provoked by bacterial tonsillitis, or by other viral upper respiratory infections; they may occur in individuals who
have already undergone tonsillectomy. It is presumed, although not proven, that the first episode represents the
onset of the disease. Patients may complain of flank pain during acute episodes, which usually reflects stretching
of the renal capsules. Low grade fever may also be present. These features can mimic urinary tract infection or
urolithiasis. Most patients have only a few episodes of visible hematuria and episodes usually recur for a few years
at most. A discussion of clinical clues that may distinguish IgA nephropathy from poststreptococcal
glomerulonephritis can be found separately. (See "Differential diagnosis and evaluation of glomerular disease",
section on 'Hematuria following upper respiratory infection' .)
2- Another 30 to 40 percent have microscopic hematuria and usually mild proteinuria, and are incidentally
detected on a routine examination. In these patients, the disease is of uncertain duration. Gross hematuria will
eventually occur in 20 to 25 percent of these patients
3- Less than 10 percent present with either nephrotic syndrome or acute rapidly progressive glomerulonephritis
picture characterized by edema, hypertension, and renal insufficiency as well as hematuria. Rarely, IgA
nephropathy may present with malignant hypertension. It is usually presumed that patients have longstanding
disease which was not detected earlier because the patient did not have visible hematuria or undergo routine
urinalysis.

IGA NEPHROPATHY VERSUS PSGN


Patients with IgA nephropathy, which is the most common cause in primary glomerulonephritis in many developed
countries, often have one or recurrent episodes of gross hematuria, usually occurring one to three days after a URI
due to either viral or bacterial infections.
Poststreptococcal glomerulonephritis (PSGN) is induced by pharyngeal or skin infection with specific nephritogenic
strains of group A, beta-hemolytic streptococci. PSGN and IgA nephropathy can be distinguished by renal biopsy,
but biopsy usually is not be performed if the clinical history is highly suggestive of PSGN and resolution begins
within one to two weeks and usually not performed in IgA nephropathy for hematuria in the absence of findings
suggestive of a worse prognosis such as proteinuria above 1000 mg/day or an elevated serum creatinine.
The latent period from the onset of infection to the detection of hematuria is between one and three weeks with
group A, beta-hemolytic streptococcal (GAS) pharyngitis (and between three and six weeks following streptococcal
skin infection) compared with less than five days in IgA nephropathy after a bacterial or viral URI.
Throat culture and laboratory tests for GAS (eg, streptozyme test) should be positive in PSGN following a URI but
usually not in IgA nephropathy. Recurrent episodes of gross hematuria are common in IgA nephropathy but rare in
PSGN since only certain streptococcal strains can cause renal disease and there is long-term persistence of
antibodies against nephritis-associated streptococcal antigen, a leading candidate for the causative antigen .
IgA nephropathy is typically a chronic disease and persistent microscopic hematuria is common after resolution of
the URI. In contrast, PSGN gradually resolves after the infection has cleared. In children, a diuresis typically begins
within one week, the serum creatinine returns to the previous baseline by three to four weeks, and hematuria
resolves within three to six months

21- post parathyroidectomy. discharged on day 4. levels of calcium was 1.17 at that time. now after 2
days presents with features of hypocalcemia. what is immediate mx? goldstar
a) calcium carbonate
b) calcium carbonate plus vit. d3
c) calcitriol
d) infusion calcium
Severe hypocalcaemia (<1.9 mmol/L and/or symptomatic at any level below reference range). This is a medical
emergency
• Administer IV calcium gluconate

22- man with bilateral knee osteoarthritis comes with left lateral leg pain during night and the pain
relieves after at least 10 min walking what is of the following will help u in reaching ur diagnosis
a) loss of sensation in left lateral leg(common proneal injury) ???
b) burger s sign
c) loss of pulsation
23-female bulimia case .. bing eating and induced vomiting , which of the following is most specific to
find in history that consist with this symptoms:
A) delayed menarch
B) sexual abuse
C) parental separation
not sure what was the other choices
Depression, anxiety, drug abuse, eating disorder >> sexual abuse in childhood

24-female at 41 weeks gestation didn't have any contractions by now, amniotomy was done and waited
progress of labour ... fetal CTG dropped to 70 for 4 minutes then got back to normal 110 ... knowing that
the cervix is 8 cm dilated and fetal head is on the level of ischial tubrosity and fully effaced occipito
posterior fetal head .. which of the following should be done
A)C. section
B) Ventouse delivery
C) fetal scalp sampling
‫ گاید الین اینو نوشته سزارین‬:rolonged bradycardia (<100 bpm for >5 minutes).
‫ االنم که خوب شده به نظرم باید‬.‫ دقیقه نیس‬5 ‫ این که شرط دوم رو نداره یعنی زیر‬scalp ‫بشه‬.

25-Ankle fracture with cold foot and impalpable pulse,next step?


Reduction

26- couple came for primary infertility, sperm 19 million, abnormal 65%,motility 40%,what advice
A)spontaneous pregnancy is not impossible
27- young lady with diarrhea with mucous and bleeding per rectum +++++glossitis …Dx
A-Crohns disease
B-Ulcerative colitis
C-Carcinoma

.‫ كرون انتخاب منطقى هست‬.‫ اما درگيرى ايلئوم ميشه كرون‬.‫البته كرون خونريزى كمتر هست اما ميتونه باشه‬
‫ شایعتر هست‬uc ‫کرون هست که بیشتر گلوسیت میده اما از طرفی هم خونریزی تو‬

28-‫ سوال ونالفاكسين ميخوايم برا طوالني مدت بديم چيو چك كنيم؟‬bun, cr/ ast alt

clinicians should caution patients to not abruptly discontinue antidepressants, which may precipitate dysphoria,
dizziness, gastrointestinal distress, fatigue, chills, and myalgias. This discontinuation syndrome appears to be
particularly common with venlafaxine and relatively uncommon with milnacipran .
Medical tests and plasma levels — No specific medical tests are required before starting SNRIs, and drug serum
concentrations are not routinely monitored because they have not been shown to correlate with clinical response .
However, levels can assess adherence and whether unresponsive patients are rapid metabolizers. Levels can also
establish that it is safe to begin another serotonergic drug (eg, a selective serotonin reuptake inhibitor or
monoamine oxidase inhibitor) after discontinuing an SNRI, in order to avoid the serotonin syndrome
Polymorphism in the hepatic cytochrome P450 enzyme CYP2D6, which converts venlafaxine to the active
metabolite desvenlafaxine , may possibly influence the efficacy of venlafaxine.and were classified as extensive or
poor metabolizers based upon the plasma concentration ratio of desvenlafaxine to venlafaxine [ 39 ]. Remission
was greater in patients who were extensive metabolizers compared with poor metabolizers (56 versus 37 percent);
the venlafaxine dose and tolerability were comparable for the two groups.
Nausea – 24 versus 16 percent
Dizziness – 13 versus 9 percent
Dry mouth – 12 versus 10 percent
Insomnia – 11 versus 11 percent
Diaphoresis – 10 versus 5 percent
Constipation – 9 versus 9 percent
Venlafaxine may increase the risk of upper gastrointestinal bleeding
Another adverse affect that can occur with venlafaxine is increased blood pressure, especially at higher doses.
venlafaxine impairs sexual function , but is weight neutral
Overdoses of venlafaxine can lead to hypertension, hypotension, cardiac arrhythmias, seizures, the serotonin
syndrome, and death . Prior to prescribing SNRIs, clinicians should discuss drug interactions, side effects, time to
response, and stopping the medication. No specific medical tests are required before starting SNRIs and drug
serum concentrations are not routinely performed. Administering SNRIs with food may reduce nausea, which is the
most common side effect.
since venlafaxin’s metabolism is hepatic checking LFT frequently is required.

29-
The major adverse effect with verapamil is constipation, occurring in over 25 percent of patients Verapamil and, to
a lesser degree, diltiazem can diminish cardiac contractility and slow cardiac conduction [ 3 ]. As a result, these
drugs are relatively contraindicated in patients who are taking beta blockers or who have severe left ventricular
systolic dysfunction, sick sinus syndrome, and second or third degree atrioventricular block

Digoxin can produce all types of AV block but mobitz thye 2 in rare

 Mobitz type I AV block, in which progressive PR interval prolongation precedes a non-conducted P wave.
 Mobitz type II AV block, in which the PR interval remains unchanged prior to a P wave that suddenly fails to
conduct to the ventricles.(2:1 or 3:1 …)
Block of a 2:1 ratio, which can be either Mobitz 2 and requires careful evaluation.
30-which of the following UNLIKELY cause fetal growth restriction
a-cmv
b- triosomy 13
c-thalassaemia minor with hb 8
d- mother with essential hypertension require methyldopa
e- lupus nephritis
IUGR causes: maternal/htn,dm,hemoglobinopathies,autoimmun disease,malnutrition,smoking,thrombophilia,high
altirude residing
Fetus/ Preeclampsia,multiple gestation,uterine malformations,Placental insufficiencies,chromosomal
abnormalities,Vertically transmitted infections

31-Patient on venlafaxine now confused and sleeping in day and wandering in night and asking foor breakfast at
night: what inv will u do:
A. KFT,
B. LFT,
C. TFT,
D. CT .
Hyponathremia
: ‫دوستان من اين ونالفاكسين رو خيلى دنبالش گشتم نتيجه بررسى هام اين شد‬
‫ بشه گفته در‬A‫ ميتونه بده و باعث هيپوناترمى‬SIADH ‫ رو باهاش چك كنيد ولى چون‬LFT ‫ كبدى داره ولى هيچ جا نديدم نوشته باشه‬A‫درسته كه متابوليسم‬
. ‫ نشن‬A‫افراد هاى ريسك كه االن گايدالينش رو ميذارم سديم سرم رو چك بكنيد كه هيپوناترميك‬

32-prophylaxis of pertusis :
chemoProphylaxis A 7-day course of the treatment antibiotics is recommended for household and other close
contacts, regardless of immunisation status, commenced within 3 weeks of onset of cough in the patient.
School exclusion is necessary until at least 5 days of antibiotic use.
Clarithromycin (child 7.5 mg/kg up to) 500 mg 12 hourly for 7 days (or) erythromycin or azithromycin
Vaccination:
 Routine immunization — Routine vaccination of children, adolescents, and adults (including pregnant
women) is the most important preventive strategy. Routine vaccination against pertussis is discussed
separately.
 Immunization of contacts – Close contacts of the index case who are younger than seven years of age and
who are unimmunized or underimmunized should have pertussis immunization initiated or continued
according to the recommended .
 Immunization after pertussis infection – Well-documented pertussis infection (eg, a positive culture for B.
pertussis or an epidemiologic link to a culture-positive case) is likely to confer immunity against pertussis.
However, the duration of immunity is unknown. The AAP recommends that children who have had well-
documented pertussis disease complete the primary diphtheria-tetanus-pertussis vaccine series with
diphtheria-tetanus-acellular pertussis vaccine (DTaP).
 Neonatal immunization – Early neonatal immunization is a strategy that is being studied, but is not yet
recommended.

32.1- Pertussis infection of child, have a sis of 24 mnth fully vaccinated and all the members of the
family treated with erythromycin, wats next best option...
Sis needs chemoprophylaxis
 We recommend booster vaccination with Tdap for adolescents and adults. Vaccination is particularly
important for adults who have close contact with infants aged younger than one year (such as
grandparents, child care providers, and healthcare providers).
 We recommend postexposure prophylaxis for individuals and healthcare personnel in close contact with
cases of pertussis ( Grade 1B ); the prophylaxis regimen is the same as the treatment regimen ( table 1 ).
Prior vaccination with Tdap should not be considered sufficiently protective to eliminate need for
chemoprophylaxis.
 Patients with B. pertussis infection should avoid contact with young children and infants until they
have completed at least five days of antibiotics. Similarly, patients working in schools, daycare
centers, or healthcare facilities should not return to work until after five days of antibiotic
treatment.

33-perthes xray give asking for definite treatment:


1.pinning
2.rest
3.non wt bearing
4.tractions
Current treatment focuses on maintaining containment of the femoral head within the acetabulum, through the
use of splints or occasionally surgery, although the data for these interventions are limited . Patients diagnosed
with LCP should be made nonweight-bearing and referred to an experienced pediatric orthopedist for
management.
The goals of treatment of SCFE are stabilization of the diseased physis to prevent further slippage and avoidance of
complications. The treatment of SCFE is operative. The gold standard for stabilization, regardless of severity, is the
use of a single cannulated screw placed in the center of the epiphysis. Stabilization usually involves pinning in situ
(ie, without attempt at reduction) because manipulation has been found to correlate with a poorer long-term
prognosis. Closed reduction of an acute slip before pinning is controversial.

34-ECG about HEART BLOCK, last night found collapse in garden, brought by her daughter to
you. Hx of heart disease,hypothyroid hx, DM, temperature was 34.3 deg. What was the cause
of his collapse?
a) Acute myocardial ischamia
b) hypothyroidism
c)hypothermia

‫ بيماران) و هيپوناترمى ميتونه از بلوك درجه يك تا بلوك كامل رو ايجاد‬٪٥٠ ‫يكى از اتفاقاتى كه توى كماى ميكزادم ميوفته هيپوناترمى هست (در‬
‫و هيپوترمى بيمار هم ميشه به همين كماى ميكزادم نسبت داد‬.‫كنه‬

Early recognition and therapy of myxedema coma are essential. Treatment should be begun on the basis
of clinical suspicion without waiting for laboratory results.
Myxedema coma is defined as severe hypothyroidism leading to decreased mental status, hypothermia, and other
symptoms related to slowing of function in multiple organs. It is a medical emergency with a high mortality rate.
Fortunately, it is now a rare presentation of hypothyroidism, likely due to earlier diagnosis as a result of the
widespread availability of thyrotropin (TSH) assays.
Myxedema coma can occur as the culmination of severe, long-standing hypothyroidism or be precipitated by an
acute event such as infection, myocardial infarction, cold exposure, or the administration of sedative drugs,
especially opioids
The hallmarks of myxedema coma are decreased mental status and hypothermia, but hypotension, bradycardia,
hyponatremia, hypoglycemia, and hypoventilation are often present as well. Puffiness of the hands and face, a
thickened nose, swollen lips, and an enlarged tongue may occur secondary to nonpitting edema with abnormal
deposits of mucin in the skin and other tissues (myxedema).
TREATMENT — Myxedema coma is an endocrine emergency and should be treated aggressively. The mortality
rate remains high at 30 to 40 percent, with elderly patients and those with cardiac complications, reduced
consciousness, persistent hypothermia, and sepsis being at greatest risk . Treatment consists of thyroid hormone,
supportive measures, and appropriate management of coexisting problems such as infection
In addition, until the possibility of coexisting adrenal insufficiency has been excluded, the patient must
be treated with glucocorticoids in stress doses (eg, hydrocortisone given intravenously, 100 mg every
eight hours)

We suggest an initial dose of 200 to 400 mcg T4 intravenously, followed by daily intravenous doses of 50 to 100
mcg until the patient can take T4 orally. Until coexisting adrenal insufficiency can be excluded, the patient should
be given high-dose glucocorticoid therapy ( hydrocortisone 100 mg intravenously every eight to twelve hours for
two days

35-Rt lower chest pain radiated to the upper abdomen…could be sth about pneumonia…O/E temp:37 ,
abdominal guarding but no rigidity on tenderness.Dx?
Lung abscess
Empyema
Subphrenic abscess
Lung abscess: necrotizing pneumonia:fever, cough, and sputum production putrid or sour-tasting sputum.A lung
abscess is typically diagnosed when a chest radiograph reveals a pulmonary infiltrate with a cavity, indicating tissue
necrosis. The only methods available for obtaining uncontaminated specimens are transtracheal aspirates (TTA),
transthoracic needle aspirates (TTNA), pleural fluid, and blood cultures. clindamycin (600 mg IV every eight hours,
followed by 150 to 300 mg orally four times daily)Other drugs: ampicillin-sulbactam 3 g IV every six hours),
penicillin plus metronidazole , or a carbapenem.We suggest continuing antibiotic treatment until the chest x-ray
shows a small, stable residual lesion or is clear. This generally requires several months of treatment.

Subphrenic abscess:Pus under a patient's diaphragm has usually spread there from somewhere else in his
abdomen. A subphrenic abscess may be secondary to: (1) Peritonitis, either local or general, following a perforated
peptic (11.2) or a typhoid ulcer (31.8), or appendicitis (12.1), or PID (6.6) or infection following Caesarean section
(18.11). (2) An injury which has ruptured a hollow viscus and contaminated his peritoneal cavity (66.2). (3) A
laparotomy during which his peritoneal cavity was contaminated (9.2). (4) A ruptured amoebic liver abscess
(31.12).
characterized by an accumulation of infected fluid between the diaphragm, the liver and the spleen.[1] This
abscess develops after surgical operations like bowel perforation or splenectomy. Presents with cough, increased
respiratory rate with shallow respiration, basal atelectasis, or pneumonia ,diminished or absent breath sounds,
hiccups, dullness in percussion, tenderness over the 8th–11th ribs, fever, chills, anorexia and shoulder tip pain on
the affected side. Lack of treatment or misdiagnosis could quickly lead to sepsis, septic shock, and death.[2] It is
also associated with peritonitis
Common clinical features of bacterial pneumonia with parapneumonic effusion include cough, fever, pleuritic
chest pain, dyspnea, and sputum production.TX: plural drnage + AB (6-8 w)

36- pic of red eye, post op case of cataract , pt woke up on day 3 lid swelling little hypopyon plus red
conjunctiva. Pupil seems small,decreased visual acuity most like DX
a. hypopion
b. acute iritis
c. glaucoma
‫ ميده و دوم‬mid-sized pupil ‫ كه اولى‬.uveitis &‫ هست و ديگرى‬acute angle closure glaucoma ‫ دو تا تشخيص مهم يكى‬red eye ‫در‬
‫ بنظرم هايپوپيون& تشخيص نيست بلكه يه يافته ى بالينى هست كه ميتونه همراه با يوئيت باشه! گلوکم زاویه بسته مردمک فیکسد دایلیتد‬.miosis
‫میده‬

Uveitis is defined as inflammation of the uveal tract, which includes the iris, ciliary body, and choroid. Anterior
uveitis, the form most likely to present emergently, includes iritis; iridocyclitis, in which the ciliary body is also
involved; and anterior cyclitis.

37-A living donor kidney transplant is assessed after OT. Doctor said, he will recover well, which ego
mechanism:
alA-truism
B-Sumlimation
C-Repression
D-Introjection
E-Reaction formation

Altruism or selflessness is the principle or practice of concern for the welfare of others more than
yourself.
Sublimation: Transformation of unhelpful emotions or instincts into healthy actions, behaviours, or
emotions, for example, playing a heavy contact sport such as football or rugby can transform aggression
into a game.

37.5- In assessment of a living kidney donor which of the following you need to exclude in the donor :
a- Altruism
b- Reaction formation
c- Regression

38-pt with past history of MI,labs given with chol =4.5, next?
A-add statin
B-reduced weight

39-.Old female with change in colour of face & neck with mild swelling , inv. Of choice ?
A. CXR
B.CT NECK
C.CT CHEST
D.arterial blood gases

‫ در هر صورت اين اگه بعد كاتتر زدن‬.‫ وقتى انجام ميديم كه بخوايم ببينيم علت ترومبوز هست يا اثر فشارى از بيرون روى وريد مثل وجود توده‬ct
‫ پس بنظرم مناسبترين و اولين قدم براى اين‬.‫ موارد شواهد غير طبيعى نشون ميده‬%70 ‫ هم تا حدود‬cxr ‫ هست و‬A‫اينجورى شده خب شك به ترومبوز‬
.‫) هست‬superior vena cavogram( ‫ گلد استانداردش آنژيوگرافى‬.cxr ‫مورد ميشه‬

‫ ميشه كمك گرفت اما بخاطر اينكه خود وناكاوا به‬A‫ گلد استاندارد هم وناكاووگرام! البته از داپلر هم در موارد شك به ترومبوز‬.‫ بعد سى تى‬.cxr ‫قدم اول‬
.‫ باشه‬choice ‫ نميتون‬،‫دليل سايه دنده ها توى سونو ديده نميشه‬

40-ankle injury open wound with displacement, cold and pale, foot next most imp?
A-debridement,
B- tetanus prophylaxis,
C-surgery

‫ حاال چه بسته و چه با جراحى‬.‫ بشه در رفتگى‬reduce ‫اول بايد‬

‫ البته واضحه كه‬.‫ مختل شده اصالح بشه‬circulation ‫ تا‬،‫ بشه‬try ‫ بايد جا اندازى‬.‫ حتى اگر امكان جراحى فورى نباشه‬،‫راجع به اون مچ پا‬
‫ بعدش جاندازى ولى زخم رو نميبنديم و ميره براى دبريدمان و‬.‫قبل جاندازى زخم و تميز ميكنيم و اگه جسم خارجى تو زخم باشه خارج ميكنيم‬
...‫جراحى‬

41-pt withpulmonary edema scenario,spitting pink frothy sputum,abg shows oxy 70,ph 7.25,next
A-cpap,(continues positive airway pressure of course it’s the last resort first is by mask)
B-iv furesmide
C-nitroglycerine

Hypoxemia must be present on minimal ventilator settings, as defined by the ratio of arterial oxygen tension to fraction of
inspired oxygen (PaO 2 /FiO 2 ). The severity of the hypoxemia defines the severity of the ARDS. Mild ARDS exists when the PaO
2 /FiO 2 is >200 mmHg, but ≤300 mmHg, on invasive or non-invasive ventilator settings that include a positive end-expiratory
pressure (PEEP) or continuous positive airway pressure (CPAP) ≥5 cm H 2 O. Moderate ARDS exists when the PaO 2 /FiO 2 is
>100 mmHg, but ≤200 mmHg, on ventilator settings that include a PEEP ≥5 cm H 2 O. And, severe ARDS exists when the PaO
2 /FiO 2 is ≤100 mmHg on ventilator settings that include a PEEP ≥5 cm H 2 O.

JM:Common risk factors/associations for ARDS include (indirectly—systemic)—sepsis, shock, trauma, burns,
drug overdose (e.g. heroin), multiple transfusions, obstetric complications (e.g. eclampsia, amniotic fluid
embolism), and many direct causes such as pulmonary aspiration, toxic gas inhalation, blast injury and
pneumonia (e.g. SARS)
Non cardiogenic pulmonary edema:
Noncardiogenic pulmonary edema is identified clinically by the presence of radiographic evidence of alveolar fluid
accumulation without hemodynamic evidence to suggest a cardiogenic etiology (ie, pulmonary artery wedge
pressure ≤18 mmHg
phentolamine, CPAP and oxygen for pulmonary Oedema(venom)

The initial management of pulmonary edema, irrespective of the type or cause, is supporting vital functions.
Therefore, if the level of consciousness is decreased it may be required to proceed to tracheal intubation and
mechanical ventilation to prevent airway compromise. Hypoxia may require supplementary oxygen, but if this is
insufficient then again mechanical ventilation may be required to prevent complications.[citation needed]
Treatment of the underlying cause is the next priority; pulmonary edema secondary to infection, for instance,
would require the administration of appropriate antibiotics
42-lung abcess scenario with ab given,next
transplural drainage,
laprotomy,drain

its not drainage its lobectomy

‫ درمان جراحی لوبکتومی هست که اگر‬.‫ شک به متاستاز) نیاز به درمان جراحی ندارند‬، ‫آبسه های ریه بجز موارد نادر ( عدم پاسخ به درمان‬
‫ بیمار ضعیف باشه و ریسک باال داشته باشه‬percutaneous ‫درناژ یا اندوسکپی میشه‬.

43-pt becoming tachpnc,palpitations on entering shopping malls ,believes he can acquire hiv by touching
doors of mall etc,dx
A-OCD??
B-panic disorder with agorophohbia,
C-hypomania

. ‫ هم بود كه جواب اون ميشه‬delusional thought ‫ بيرون نميره قبالً هم بوده و توى گزينه هاش‬HIV ‫اون خانمى كه از ترس گرفتن‬
‫اين كه مريض ميگه برم بيرون ايدز بگيرم ديلوژن هست نه آگورافوبيا و امثالهم‬

44-pt do excess exercise bmi 19,checks in mirror several times etc ,dx
A-anorexia nervosa,
B-bdd,hypomania,
C-ocd

Refusal to maintain normal body weight at or above a minimum normal weight for age and height (loss to <85% of
expected body weight)

‫ اوال نوشته هیچ معیار دقیقی برای‬BMI ‫ حداقل ذکر نشده و توی‬DSM 5 ‫ که اخرین گاید الین هم هست حداقل نرمال‬bmi ‫ولی بازم‬18.5 ‫رو نوشته‬
‫گفته معیاری واسه تشخیص انورکسا نست‬

Anorexia nervosa has 2 sub types:


1- Restricting subtype (lose wt by fasting and/or hyper exercising)
2- Binge-eating/purging subtype (although it has overlapping features with bulimia nervosa, the key difference is
that pts with anorexia maintain a LOW BMI)

45-rash on face and hand. pain in both hands. rf=4 (normal =4) ,ana=574 (normal=7), rest of labs were
all normal what u ll find on xray on hand?
A-chondrocalcinosis
B-periarticular erosions
C-punched out marging.

• SLE Arthritis (non-erosive arthritis in ≥2 peripheral joints, non-decreased joint space)

Arthritis — Joint symptoms occur in over 90 percent of patients at some time during the illness and are
often the earliest manifestation [ 12 ]. Arthritis, with inflammation, occurs in 65 to 70 percent of patients
and tends to be migratory and symmetrical. Only a few joints are usually affected, especially those of the
hands. The arthritis is moderately painful and is rarely deforming

46-previous hx of dvt. now venous discoloration of ankle and calf pain. thrombosis has extend to
popliteal vein. what to give
A-compression stocking
B-warfarin

sc enoxaprin , prolonge treatment 》》 warfarin


next inv:dopler

TREATMENT OF DVT & PE FROM GUIDELINE


DVT
Compression stockings
Graduated compression stockings reduce the incidence and severity of the post-thrombotic syndrome and should
be used in all patients with deep vein thrombosis (DVT).
Anticoagulation: before starting should check APTT, INR, PLT count. options are UHF, LMWH, fondaparinux
Initial anticoagulation should be with parenteral drugs. LMWH (dalteparin, enoxaparin) and fondaparinux
have the advantages of not requiring routine laboratory monitoring.
1 dalteparin 200 units/kg up to 18 000 units SC, daily or dalteparin 100 units/kg up to 9000 units SC, twice daily
OR
1 enoxaparin 1.5 mg/kg SC, daily or enoxaparin 1 mg/kg SC, twice daily
OR
2 fondaparinux (patients less than 50 kg: 5 mg; 50 to 100 kg: 7.5 mg; more than 100 kg: 10 mg) SC, daily.
[Note: Both enoxaparin and dalteparin require dose adjustment in the presence of renal impairment GFR<30]
[Note: Fondaparinux use is not recommended in patients with renal impairment]

Intravenous UFH requires intensive laboratory monitoring and is usually used to allow rapid reversal of
anticoagulation when a high risk of active bleeding is present.
unfractionated heparin 330 units/kg loading dose SC, followed by 250 units/kg SC, twice daily.
Oral anticoagulation may be started on the same day as parenteral therapy; use:
warfarin orally, daily according to Table 3.27, then daily dosing adjusted to a target INR of 2 to 3.

Pulmonary Embolism
Stable patients: The mainstay of treatment of pulmonary embolism without haemodynamic compromise is
supportive medical care (particularly oxygen and analgesia) and therapeutic anticoagulation. anticoagulation
therapy is as mentioned above.
Unstable patients: Patients with sustained hypotension of below 90 to 100 mm Hg, support with oxygen at high
flow rates and analgesia. Therapeutic anticoagulation is required and fibrinolytic therapy may be of benefit in
patients with acceptable risk of bleeding complications.
•alteplase (patients 65 kg or more) 10 mg IV bolus, followed by 90 mg IV infusion over 2 hours.
Anticoagulation with unfractionated heparin (UFH) via an infusion should be started, as the efficacy of low
molecular weight heparins (LMWH) in this situation is unknown.
•unfractionated heparin 80 units/kg loading dose IV, followed by 18 units/kg/hour IV infusion, adjusted
according to APTT.
The UFH should be continued for a minimum of 5 days and until the INR has been above 2 on 2 consecutive days.

In conclusion: Acute DVTLMWH or fondaparinux is preferred to UHF


Acute PE stable:LMWH or Fondaparinux is pref to UHF
Unstable: UHF +/- fibrinolytic
LMWH can be used in renal impairment(GFR<30)with dose adjustment but fondaparinux cant
UHF can be used in RF with APTT monitoring.

47--.pt with bleeding dudnl ulcer margins seen on endoscopy, next step in management
Endscpc adrenaline,
suturing with laprotomy,
ppi, triple therapy

APPROACH TO THE PATIENT — The initial evaluation of a patient with upper gastrointestinal (UGI) bleeding starts
with assessing hemodynamic stability and determining the need for fluid resuscitation and/or blood transfusion.
This part of the evaluation is discussed in detail elsewhere.
Patients with clinically significant UGI bleeding (ie, signs of active UGI bleeding including hematemesis, melena, or
hematochezia, with or without hemodynamic instability or blood transfusion requirement) should be started on an
intravenous proton pump inhibitor while undergoing their initial evaluation. Once the patient is stabilized,
endoscopy is performed to diagnose high-risk lesions . Ulcers that are actively bleeding and most nonbleeding
ulcers that are at high-risk for recurrent bleeding based upon the presence of stigmata of recent hemorrhage
require endoscopic therapy. Ulcers that lack high-risk stigmata can be managed acutely with acid suppression
alone.
First of all plz NPO the patient
The appearance of ulcers can be described using the Forrest classification :

Class Ia – Spurting hemorrhage


Class Ib – Oozing hemorrhage
Class IIa – Nonbleeding visible vessel
Class IIb – Adherent clot
Class IIc – Flat pigmented spot
Class III – Clean ulcer base ENDOSCOPIC THERAPY
— Endoscopic therapy is indicated for the treatment of most ulcers with stigmata of recent hemorrhage that
increase the risk of recurrent bleeding. With appropriate treatment, high-risk lesions have recurrent bleeding rates
of 5 to 20 percent, depending upon the endoscopic appearance of the ulcer. On the other hand, ulcers with a clean
base or a flat pigmented spot are at low risk of recurrent bleeding (3 to 5 percent for clean-based ulcers and 7 to
10 percent for ulcers with a flat spot) and should not be treated endoscopically
Injection therapy — Injection therapy should be used in conjunction with other forms of therapy, such as thermal
coagulation or hemoclip placement. Injection therapy should not be used as monotherapy because it is associated
with higher rates of recurrent bleeding than treatment with thermal coagulation, hemoclip placement, or
combination therapy
Injection therapy with dilute epinephrine results in local tamponade and vasospasm The technique is inexpensive
and effective for temporary hemostasis
Acid suppression — Treatment with PPIs leads to elevation of gastric pH levels, which stabilizes blood clots and
improves clinical outcomes . As a result, PPIs are recommended for all patients with peptic ulcer bleeding. Prior to
endoscopy, patients are typically started on a continuous infusion of an intravenous PPI. Twice daily dosing of an
oral proton pump inhibitor may be a reasonable alternative if intravenous formulations are not available

48-cxr of pneumonia with scenario pt cough haemoptysis, normal oxy sat, trachea central, not in much
distress but dull percussion on left lower lobe.
A-Amoxyclav, (mild)
B- procaine penicillin, (moderate)
C-vanco,
D-ceftrxne

PNEUMONIA
JM P472
Community-acquired pneumonia in people who are not or have not been in hospital recently.
A)Typical pneumonia: Streptococcus pneumoniae (majority), Haemophilus influenza.
• Often history of viral respiratory infection
• Rapidly ill with high temperature, dry cough, pleuritic pain, rigors or night sweats
• 1–2 days later may be rusty-coloured sputum
• Rapid and shallow breathing follows
• X-ray and examination: focal chest signs, consolidation
B) The atypical pneumonias: mycoplasma,legionella,coxiella,chlamydia pneumon
• Fever, malaise
• Headache
• Minimal respiratory symptoms, non-productive cough
• Signs of consolidation absent
• Chest X-ray (diffuse infiltration) incompatible with chest signs
Antibiotic treatment
Mild pneumonia: amoxycillin/clavulanate 875/125 mg (o) 12 hourly for 7 days especially if S. pneumonia
isolated or suspected plus(especially if atypical pneumonia suspected) roxithromycin 300 mg (o) daily for
7 days
Moderately severe pneumonia
This requires hospitalisation
• Neonates
• Age over 65 years
• Coexisting illness
• High temperature: >38 ° C
• Clinical features of severe pneumonia
• Involvement of more than one lobe
• Inability to tolerate oral therapy
Benzylpenicillin 1.2 g IV 4–6 hourly for 7 days or Procaine penicillin 1.5 g IM daily (drugs of choice
for S. pneumoniae )or Ceftriaxone 1 g IV daily for 7 days (in penicillin-allergic patient)
• If atypical pneumonia use doxycycline, erythromycin or roxithromycin
severe pneumonia
azithromycin 500 mg IV daily (covers Mycoplasma, Chlamydia and Legionella) plus cefotaxime 1 g IV 8 hourly
or ceftriaxone 1 g IV daily.

Pneumonia in children
Clinical features
• Tachypnoea, expiratory grunt
• Possible focal chest signs
• Diagnosis often only made by chest X-ray
Pathogens
• Viruses are the most common cause in infants.
• Mycoplasma are common in children over 5 years.
• S. pneumoniae is a cause in all age groups.
Treatment
Almost all those under 48 months should be admitted to hospital.
Mild (general guidelines only): amoxycillin (o) or roxithromycin (o)
Moderate: benzylpenicillin IV + roxithromycin (o)
Severe: flucloxacillin IV + gentamicin IV ± azithromycin (o)
49-colles fx. how to perform cast ?
-full ext
-full flex
-semi flex

jm:1469
‫ روز بعد عكس بگيريد‬١٤-١٠ ‫مورتاگ صفحه گفته‬
‫ هفته‬۳ ‫مداسکپ رو هم دیدم نوشته هفتگی چک شن تا‬

50-man with bilateral knee osteoarthritis comes with left lateral leg pain during night and the pain
relieves after at least 10 min walking what is of the following will help u in reaching ur diagnosis
a) loss of sensation in left lateral leg
b) burger s sign
c) loss of pulsation

.a ‫ جواب هم‬.Nerve entrapment A.‫نوروپاتيك‬


.‫ايسكميك بود با فعاليت بيشتر ميشد‬

51-Asperger boy 13 years old, had multiple phobias. Given sertraline, he improves and parents are
happy that he is confident and doing well now. Lately became aggressive and irritated. Reason?
1. Puberty realted mood change
2. Sertaline induced
3. Aspergens turning to ADHD
4. Relapse of Aspergens

52-painless hematuria in 45 y/o man .whats the most likely diagnosis?


A-carcinoma bladder
B- carcinoma kidney
JM 808 :
Common urological cancers that cause haematuria are the bladder (70%), kidney (17%), kidney pelvis or
ureter (7%) and prostate (5%).

‫ بهترين سيستوسكوپى‬، UA/UC ‫اولين اقدام‬

) ‫( کنسر مثانه‬.‫ هست‬upper tract ‫ گلد برای‬ivu ‫سی تی‬

‫ در تمام مواردى كه هماچورى با منشا‬،‫ هماچورى بدون درد در اين سن بايد اول كنسر مثانه رد بشه‬lower ‫ بايد سيستوسكپى انجام بشه‬،‫داريم‬

JM865 First: u/a u/c + cbc bun cr cyctoscopy in all patients is advisable. IVU and IVP are key investigations

53- A child that goes to day care presents with a dry cough. Investigation:
A. Nasopharyngeal aspirate
B. Sputum culture
C. Chest xray

CXR

54- ‫بین مریض‬،‫بار قبال هم بدون پول ویزیت اومده برخوردتون چیه‬4،‫مریض اومده با گلودرد پول ویزیت نمیخواد بده‬
‫یا اینکه تا همه ویزیتاشو نده نمیشه‬،‫بگیرچد تا پول نده نمیشه‬،‫ بیمارستان‬A‫بفرستید‬،‫ویزیت کنید‬

Violence against health professionals has increased. The ultimate sanction against violent and abusive patients is to
withhold treatment.

In what circumstances could this be considered a justifiable limitation of the doctor’s duty of care? A doctor’s duty
of care is not limitless and may be restricted by competing duties to other patients (time and resource
implications), duties to self (not to go beyond boundaries of competence and therefore to be exposed to potential
liability, not to undertake obligations where there is a recognized conscientious objection) and duties to
colleagues. An aggressive patient, who, having been informed that treatment will not be provided if such
behaviour continues, may be regarded as having made an autonomous choice to forfeit treatment in such
circumstances. Mentally ill patients and those who are not competent are not considered able to make an
autonomous choice to forfeit treatment, and therefore treatment must be provided (and reasonable force can be
used to effect necessary treatment).A doctor’s duty to act in the best interests of a patient may be balanced
against the harms to others. Except in extreme circumstances, the harms of not providing emergency treatment
will outweigh the harms of violence and aggression to healthcare professionals and other patients. This may not be
so if there is a serious threat of violence, e.g. a patient carrying a gun or knife. A National Health Service (NHS) trust
has an obligation to provide a safe working envir- onment for its employees, and it may be failing in this duty if it
does not have appropri- ate safeguards against abusive and violent patients. Trusts have policies on treating
violent patients and when it may be acceptable to refuse to continue to treat. Professional guidance recognizes
that ‘in rare circumstances’ the relationship of trust between a doc- tor and patient breaks down, perhaps by the
patient’s violent behaviour, and it may be necessary to end the relationship (Good Medical Practice. London:
General Medical Council, 2006).Doctors should be able to justify their actions and decisions.• All acts of violence,
verbal aggression and physical threats should be reported to security, and if necessary, the police.

55- ‫میخواد درآینده نزدیک باردار بشه توصیه‬،‫تا استاندارد میخوره‬2‫فرد روزی‬،‫مشاوره قبل بارداری راجع به مصرف الکل‬
‫روزی یه استاندارد‬،‫ به دو روز در هفته‬A‫محدود‬،‫االن سریعا قطع کنه‬،‫بمحص اطمینان از بارداری قطع کنه‬،‫شما چیه‬
Jm p 1216 for women who are pregnant or planning a pregnancy not drinking is the safest option

56- 22 y man request for prostatic cancer screening, what will you do?
Do screening

57-

‫و‬،‫معاینه قرمزی وولوووازینال‬،‫اخیرا از دیسپارونی با پارتنر چند ساله شاکی‬،‫دیسکامفورت وولوواژینیت مزمن‬
‫برداشتن ضایعه‬،‫درمان موضعی کورتون‬،‫پی سی آر هرپس‬،‫بیوپسی‬،‫چیکار میکنید‬...،‫زخم‬

58-a patient comes to ur clinic,aggrasive with staff,comes for non emergncy issues in emergency,havnt
paid previous bills,next step?
a)ask fees before consultation
b)see him in b/w other appointments??
c)give him appointment of tomorrow
d)send him to nearest hiospital
‫‪59-Patient came with sore throat complain. He hasn't made appointment and hasn't paied his former‬‬
‫?‪bills, wants to see you(doctor).What do you do‬‬
‫?? ‪a. Refer to the nearest hospital‬‬
‫??‪b. Visit him between other patients‬‬
‫‪c. Say to him you'll visit him after paying bills‬‬
‫‪d. Refuse to see him‬‬

‫ز ‪60-Testicular swelling with no other complaint and all other examinations Nl.‬‬
‫‪1-AFP‬‬
‫‪2-Review in 3 mo‬‬

‫در معاينه يك توده روى بيضه لمس ميشه ‪ ،‬خود بيضه نرمال هست و وازدفران هم به آزادى قابل لمسه ‪ .‬سونوگرافى هم‬
‫‪.‬يك توده كيستيك نشون داده‬
‫‪ .‬اين توصيف يعنى كيست اپيديديم هست‬
‫‪ .‬كيستهاى اپيديديم تقريبا ً همه شون خوش خيم هستن و نيازى به چك تومورماركر نيست‬
‫‪.‬ولى كيستهاى بيضه ‪ ٪٢٣‬بدخيم هستن و حتما ً تومورماركر بايد چك بشه‬
‫‪ .‬البته اگه مريض به لحاظ كاسمتيك با كيستش مشكل داشته باشه به شيوه اسكروتال اكسازش ميكنيم‬
‫‪ .‬ولى اين همچين چيزى نگفته بود ‪ .‬گفته بود خودش مشكلى با توده نداره ولى زنش چون نگران بوده آورددش دكتر‬
‫كيست بيضه از اينگواينال اپروچ ميشه ‪ ،‬اپيديديم از اسكروتال‬
61-Multiple medication use with low MCV.
1- Aspirin,
2- Diclofenac,
3- Hydroxychloroquine

‫ از بين‬NSAID ‫ مابقى بيشتر هست نسبت به آسپيرين‬، ‫ ها فقط ايبوپروفن هست كه عوارض گوارشيش از آسپيرين كمتره‬.
‫لذا اين تست ميشه ديكلوفناك‬
Diclofenac > ASA> iboprofen

62-‫کدوم تو بارداری تاثیر بدتری داره؟‬


۱)‫هرویین‬
۲)‫ماری جوانا‬
۳)‫دیازپام‬
۴‫) الکل‬

63- Post operative case after some GI surgery, ECG shown with hyperkalemic changes, tall tented T
waves. 0.9% N.saline was being infused at 60ml/hour and his urine output was 15 ml in 3 hours. What is
next best step in management?
a) increase I/V infusion rate**********
b) give calcium chloride
c) change to Hartman solution
d) check electrolytes
e) give insulin

، ‫ وقتى تغيير‬ECG ‫داده يعنى داره رو قلب اثر ميكنه و كلسيم ميديم كه جلوى اين اثرات رو بگيريم تا اينكه پتاسيم اصالح‬
‫بشه‬.

64-. Pt with abd discomfort for 6 mo. u/s shows 10 cm cystic lesion in epigastric area . best option of Rx?
A) endoscopic gastrostomy
B) laparotomy
C) percutaneous drainage
D) drainage by ERCP
‫ اينجا‬best option ‫ اگر كيست مرتبط با مجرا باشه و انسداد در مجرا هم داشته‬،‫بستگى به اين داره كه كيست كجا باشه‬
‫ باشيم‬ercp ‫اما در غير اين صورت بهترين روش معموالً اندوسكوپيك گاستروستومى هست‬.
Infected <<< percutaneous drainage

65-A woman with dizziness and shoulder pain with ecg showing bradycardia with p-r interval 5 small
squares . asking the management :
a- Pacemaker
b- TFT
c- Atropine

First degree block is more than 20 milisecond, so the patient dose not requier pacemaker or atropine.
More probable causes : hyperkalemia, hypermagnesia,adrenal insufficiency, hypothyroidy.

66-A 16yrs old boy has come with the compliants of recent decrease of school performance, 3weeks ago
he broke up with her girlfriend. He is suffering from lack of energy, sleep disturbance, feel like crying for
few months.What will be the most appropriate initial step of Mx?
A.Empathy about the break up
B.CBT
C.Fluxetine
D.Fluvoxamine.

Supportive psychotherapy provides a “holding environment,” meaning that treatment is marked by emotional
safety, trust, consistency, and structure [15,17,36]. This promotes corrective emotional experiences, in which the
patient learns to tolerate situations that were previously intolerable. The clinician expresses empathy to
corroborate the emotional experience of the patient [17]. The clinician also conveys interest, respect, acceptance,
and approval of the patient to promote a positive therapeutic alliance. This helps the patient to identify with the
clinician, become more amenable to specific interventions, and predicts a positive outcome [5,37]. In addition, the
clinician solicits feedback from the patient to ensure that the clinician correctly understands what the patient says.
The therapeutic alliance is discussed further within the context of psychodynamic psychotherapy.

67-45yr male,FOBT negative 2weeks ago.father was diagnosed of ca colon at 57yr.maternal aunt was
diagnosed at 61.next what to do
FOBT 2yearly
Colonoscopy yearly
Sigmoidoscopy yearly
Nothing
Plan:2yrly FBOT + 5 yrly flex sigmo from 47

COLORECTAL CA SCREENING
NHMRC:
Normal people: FOBT 2 yrly
Slight increased risk: one first degree relative with coc at 55 or older/two relatives with coc at 55 or
older diff sidesFOBT 2yely + flexible sigmoidoscopy 5 yrly from 50 or 10 year earlier of
the onset of cancer in relatives(whichever comes first)
Moderate increased: one first degree with coc below 55 / 2 first degree with coc at same side at any
ageFOBT 2yrly + colonoscopy 5 yrly from 10 year younger than relatives
High risk:three or more relatives with coc on the same side, 2 or more same side with coc before 50
FAP sigmoidoscopy from 12-15 to 35 then 5 yrly / HNPCC 2yrly colonoscopy from 25

68- 4 years old child with intermittent pain in the abdomen with vomiting which been for 12 months on
examination it s all normal what will lead u to diagnosis?
a-erect and supine xray (if gastrointestinal origin is more suspicious)
b-barium enema
c-us during the attack (if genitourinary origin is more suspicious)
d-gastrographin and follow through

68.1_ pt with abdominal pain he is 4 years old , pain reccurent for last 2 years , last attack was in left
flank inv:
a- us during attack
b- micturating cystogram
c- erect and supine xray
d- small intestine meal and follow
e- electrolytes and urea , creatinine

68.2_A 4 y.o. boy comes with intermittent abdominal pain. Usually it lasts for 12 hours, sometimes he is
vomiting during this time, then the pain goes away spontaneously. O/e no abnormalities mentioned.
Today he is also complaining of the left flank pain. WOF will help you in dx?

a. CT abdomen
b. Erect and supine X-ray of abdomen
c. USG abdomen during the episode
d. Micturition cystogram
e. Barium meal and follow through

68.3_a 4 years old child with intermittent pain in the abdomen with vomiting which been for 12 months on
examination it s all normal what will lead u to diagnosis
a-erect and supine xray
b-barium enema
c-us during the attack
d-gadtrographin and follow through

very challenging but conclusion: according to JM p341 there is no need of investigations if u can link the scenario
with functional abd pain otherwise its sth that reoccurs and examinations are normal thus a diagnostic modality
during attack is needed. Not to mention malignancies won’t follow intermittent pain they will have increasing
pattern of pain and other symptoms while examinations are not normal. Since the problem could be with either
gastrointestinal sys (malrotation,strings,adhisions and any other problem that cause intermittent obstruction) or
genitourinary sys (intermittent infections like cystitis due to reflux , pyelonephritis,…) :
A simple FBE looking for leukocytosis + u/a + MCU would be the first option .
Depending on the exact location of pain a simple supine and erect x-ray is the first option in approach to most
obstructions.
Usg will be helpful in evaluation of urinary sys problems like dilated urethra or nephrolithiasis.
According to JM second inv if the plain x-ray in nondiagnostic for obstruction is gastrographin follow through.

69- picture of dupuytranes contracture .before giving t/m appropriate initial investigation
A-Usg
B-Mri
C-Blood glucose

69.1_Pic of duputren’s contraction. The man works with a drill and has 2 beers for a day. What is the most
possible cause?
-alcohol
-drill
-diabetes
-cirrhosis

69.2_Pic Dupuytren and the case; normal glucose which investigation before tx
FBS
LFT
US hand (before surgery or injection)

69.3_Dupuytren contractors most cause ?


Alcohol
Smoking
.DM
AD

JMp720: The cause is unknown but there is an AD genetic predisposition. It is associated with smoking, alcoholism,
liver cirrhosis, COPD, diabetes and heavy manual labour.

Uptodate: Associated conditions include repetitive hand use and vibratory trauma; diabetes mellitus, complex
regional pain syndrome; the presence of other localized fibrosing conditions; and cigarette smoking and alcohol
consumption. The diagnosis of Dupuytren’s contracture is clinical, based upon the history of painless stiffness of
the fingers and upon the presence of characteristic findings on physical examination. The goals of treatment are to
improve flexibility of the flexor tendons and to evaluate the need for surgery or other interventions.Patients with
mild symptoms should be instructed to passively stretch The standard treatments for moderate to severe flexion
contractures are surgical (open fasciectomy), as well as percutaneous or open fasciotomy or needle
aponeurotomy.

Medscype: No routine laboratory but FBS if dm is suggested


No routine imaging but usg helps prior to surgery or intralesional injection

70-a pt presents with pain in foot...has low capilary refill....poor pedal pulses...a ulcer on med
malleolus..what is reason for pain?
1.low capillary refill
2.poor pedal pulses
3.ulcer on med malleolus

71- Women 24 years old, on all possible drugs available!!Which one is most teratogenic?
A-Heroin
B-Alcohol
C-Cocain
D-Marijuana
E-Smoking
cocaine is not associated with congenital anomalies, its use is associated with increased risk of fetal intracranial
hemorrhage .alcohol use is associated with fetal alcohol syndrome (more than 12 standard drinks (120gr) per day).
With only 5 to 7 standard drinks per day.Heroine and other opiates can be associated with the following
complications:- Low birth weight usually associate with intrauterine growth restriction(IUGR)- Preterm labour-
Drug withdrawal for the neonate (neonate abstinence syndrome)- Increased risk of sudden infant death syndrome
(SIDS).Amphetamines are not associated with congenital neurological anomalies, but their complications include:-
Intrauterine growth restriction (IUGR)- Preterm delivery- Increased neonatal mortality . Bear in mind that
malformations are different from perinatal complications. For example, smoking is associated with several
perinatal risks, but there are no congenital malformations associated with it.

72--40 yo female , history of migraine , polycystic kidney ,sudden occipital headache , no neck stiffeness
nor photophopia , ct normaland 2 LP unsuccessful , next appropriate inv. ?
-repeat ct
-MRI
-cerebral angio
•There appears to be an increased risk of stroke associated with migraine, particularly in women who have
migraine with aura. However, the absolute increase in the risk of stroke is small.
•The major extra renal complications of ADPKD include cerebral aneurysms, hepatic and pancreatic cysts, cardiac
valve disease, colonic diverticula, and abdominal wall and inguinal hernias and the risk of aneurism

73- A patient with rheumatoid arthritis got a flare up and doctor wants to administer azathioprine.
which screening test should be done before starting this drug?
A-Tb Gold test
B-Thiopurinemethyltransferase genotype screening***
C-Thiopurine methyl transferase phenotype screening

‫ تو آپتودیت دوتاشو نوشته ولی تو دیویدسون نوشته االن به صورت روتین انزیم‬TPMT ‫قبل از تجویز آزیترو اندازه گیری‬
‫ پس فنوتیپ درسته‬،‫میشه‬.
Expert opinions differ regarding the role of TPMT genotyping prior to the administration of thiopurines for
treatment of inflammatory and autoimmune disorders. Some advocate routine testing while others, citing the low
frequency of homozygous variants among Caucasians and the fact that the majority of patients who develop
myelosuppression while taking AZA do not have detectable TPMT gene mutations, disagree with this approach.
Variation in the TPMT gene can result in functional inactivation of the enzyme and in a markedly increased risk of
life-threatening myelosuppression. For this reason, TPMT testing is recommended by the US Food and Drug
Administration (FDA) prior to treatment with a thiopurine.
In the case of azathioprine, they recommend that "consideration be given to either genotype or phenotype
patients for TPMT." For individuals with low or absent TPMT activity, dose reductions of up to 90 percent may be
needed, based upon experience in pediatric acute lymphocytic leukemia [58-60]. Specific dosing recommendations
for thiopurines according to TPMT phenotype were made by the Clinical Pharmacogenetics Implementation
Consortium in 2011 [61] and reiterated in their 2013 update
Prospective testing of TPMT genotype for dose adjustments has been shown to be effective at reducing toxicity of
6-MP without compromising efficacy.
Expert opinions differ regarding the role of TPMT genotyping prior to the administration of thiopurines for
treatment of inflammatory and autoimmune disorders; some advocate routine testing while others, citing the low
frequency of homozygous variants among Caucasians (only about 1 in 300) and the fact that the majority of
patients who develop myelosuppression while taking azathioprine do not have detectable TPMT gene mutations,
disagree with this approach. Some clinicians, in particular those treating acute leukemia with 6-MP, only perform
TPMT genotyping if there is unexpectedly severe or prolonged myelosuppression.

74-60 year old man complaining of loin and groin pain with 3 previous episodes. Previous stones were ca
oxalate. Best method to prevent recurrence?
1-Allopurinol
2- Ca free diet
3- Na restriction
4- Hydrochlorothiazide

‫ درمانى هست كه اگرچه توصيه ميشه ولى‬١ ‫ گزينه‬،‫ كالً باعث افزايش كلسيم ادرار ميشه‬٢ ‫ گزينه‬controversy ‫ محدود كردن سديم‬.‫هست‬
‫باعث افزايش بازجذب كلسيم و در نتيجه كاهش كلسيم ادرار ميشه و ميتونه موثر باشه ولى از طرفى مصرف سديم باعث افزايش حجم ادرار‬
‫ دركل آپتوديت نوشته كه اثر مستقيم كاهش سديم مصرفى بر روى تشكيل سنگ‬.‫ميشه كه عامل مهمى در پيشگيرى از تشكيل سنگ هست‬
unclear ‫ تجويز دارو در موارد عود و تشكيل مجدد سنگهاى جديد يا‬،٤ ‫ اما گزينه‬.‫ اگرچه توصيه به محدود كردن مصرف سديم شده‬،‫هست‬
‫ آپتوديت ميگه كه نشون داده شده كه تجويز تيازيد اثر‬.‫ انديكاسيون داره‬،‫افزايش سايز سنگهاى قبلى و جواب ندادن به درمان با رژيم غذايى‬
significant ‫در كاهش ميزان عود در مقايسه با كسانى كه بدون دارو درمان ميشدن داره‬.

75-a 27 yrs. old lady with pelvic abscess scenario , breast feeding her baby and has allergy to penicillin,
what to give?
a. Vancomycin +gentamycin +metronidazole (with metro, a single dose if needed, breast feeding should be
hold for 24 h otherwise its contraindicated)
b. Ampicillin +metronidazole
c. Doxy+ metronidazole
d. Ceftriaxone
e. Cephazolin
A tuboovarian abscess (TOA) is an inflammatory mass involving the fallopian tube, ovary and, occasionally, other
adjacent pelvic organs (eg, bowel, bladder).Mixtures of aerobic, facultative, and anaerobic organisms have been
isolated from TOAsThe classic presentation of TOA includes acute lower abdominal pain, fever, chills, and vaginal
discharge. However, the presentation of many women with TOA differs from this classic scenario.TOAs are most
commonly detected based on findings from pelvic imaging. Ultrasound is typically the first line imaging .Women
with a ruptured TOA classically present with an acute abdomen and signs of sepsis, although this is not invariably
noted. We suggest treatment with antibiotic therapy alone for women who meet the following criteria:
hemodynamically stable with no signs of a ruptured TOA; abscess <9 cm in diameter; adequate response to
antibiotic therapy; and premenopausal . For women who show no improvement on antibiotic therapy alone, but
are not worsening, we suggest a minimally invasive abscess drainage procedure. All regimens share the common
characteristics of broad coverage for all associated bacteria (similar to treatment regimens for pelvic inflammatory
disease [PID]), including coverage for the sexually-transmitted pathogens ( N. gonorrhoeae and C. trachomatis ,
although these bacteria are rarely isolated from a TOA)
TX: uptodate:Gr+ Gr- and anaerobics should be covered: cefotetan+doxy/clinda+genta/ampi-sulbac+doxy
JMp1141: sexually transmitted:ceftri+azithro+metro
Nonsexually(genital manipulation):co-amoxi+azithro/ co-amoxi+genta+metro

TETRACYCLINES: Tetracyclines are broad-spectrum bacteriostatic antibiotics that inhibit protein synthesis. They are
active against many gram-positive and gram-negative bacteria, including anaerobes, rickettsiae, chlamydiae,
mycoplasmas, and L forms; and against some protozoa, eg, amebas. The antibacterial activities of most
tetracyclines are similar except that tetracycline-resistant strains may be susceptible to doxycycline, minocycline,
and tigecycline, all of which are poor substrates for the efflux pump that mediates resistance. Differences in clinical
efficacy for susceptible organisms are minor and attributable largely to features of absorption, distribution, and
excretion of individual drugs.

Best would be: clinda+genta+/-Azithro

76- a lady in 22 week of pregnancy, came with fever, and abdominal pain in right quadrant and nausea,
she has a history of appendectomy, what is the likely diagnosis?
a. Ectopic pregnancy
b. Abortion
c. Red degeneration***( fibromi ke hemorragic shodeh..)
d. Corpus luteal cyst
e. Ovarian torsion
OVARIAN TORSION
refers to the complete or partial rotation of the ovary on its ligamentous supports, often resulting in impedance of
its blood supply. It is one of the most common gynecologic emergencies and may affect females of all ages . The
fallopian tube often twists along with the ovary; when this occurs, it is referred to as adnexal torsion. Prompt
diagnosis is important to preserve ovarian and/or tubal function and to prevent other associated morbidity.
However, making the diagnosis can be challenging because the symptoms are relatively nonspecific.
In adults, an ovarian physiologic cyst (functional cyst, corpus luteum) or a neoplasm is the most likely factor to
predispose to ovarian torsion .
Ovarian torsion may also occur in patients with normal ovaries (no mass and not enlarged).
The right ovary appears to be more likely to torse than the left, possibly because the right utero-ovarian ligament is
longer than the left and/or that the presence of the sigmoid colon in the left side of the colon may help to prevent
torsion
Risk factors :
Ovarian mass, Size, Benign versus malignant masses — Torsion is more likely to occur with benign cysts or
neoplasms rather than malignant lesions,
Normal ovaries — It is important to note that torsion may occur in the presence of normal ovaries, particularly in
the pediatric
Reproductive age — The majority of cases of ovarian torsion occur in women of reproductive age
Pregnancy — Pregnancy is associated with an increased risk of ovarian torsion.
Ovulation induction — Ovulation induction for treatment of infertility results in the formation of large ovarian
follicular cysts
Clinical presentation :
Pelvic pain (90 percent)
Adnexal mass (86 to 95 percent)
Nausea and vomiting (47 to 70 percent)
Fever (2 to 20 percent)
Abnormal genital tract bleeding (4 percent)

Fever may be a marker of adnexal necrosis, particularly in the setting of leukocytosis.


Pregnant women with torsion present in a similar fashion with lower abdominal pain, nausea, vomiting, and
possibly a low grade fever, leukocytosis, or a palpable mass
Imaging studies — We suggest pelvic ultrasound as the first line imaging study for patients with suspected ovarian
torsion. Pelvic magnetic resonance imaging (MRI) or computed tomography (CT) scan are not usually used for the
diagnosis of adnexal torsion.
MANAGEMENT — The mainstay of treatment of ovarian torsion is swift operative evaluation to preserve ovarian
function and prevent other adverse effects (eg, hemorrhage, peritonitis, adhesion formation). For most
premenopausal patients with ovarian torsion and an ovary that appears potentially viable on direct visualization,
we recommend detorsion and ovarian conservation rather than salpingo-oophorectomy. Ovarian cystectomy is
often performed if a benign mass is present. Patients with an apparently necrotic ovary or an ovarian mass that is
suspicious for malignancy require salpingo-oophorectomy. Salpingo-oophorectomy is also reasonable for
postmenopausal women.
UTERINE LEIOMYOMAS (FIBROIDS OR MYOMAS)
Symptoms attributable to uterine myomas can generally be classified into three distinct categories:
•abnormal uterine bleeding
•pelvic pressure and pain
•reproductive dysfunction
Dx : Transvaginal ultrasound is the most widely used imaging modality for evaluating fibroids due to its availability
and cost-effectiveness.

Relief of symptoms related to fibroids usually occurs at the time of menopause, when menstrual cyclicity stops and
steroid hormone levels wane. Most, but not all, women have shrinkage of leiomyomas at menopause. Use of
postmenopausal hormone therapy may cause some women with leiomyomas to continue to have symptoms after
menopause. Hormone therapy may be associated with an increase in size of existing myomas, but not with the
development of new myomas.
In pregnancy
•Most fibroids do not exhibit a significant change in volume during pregnancy; those that do increase in size
tend to do so primarily in the first trimester
•Fibroid pain likely results from decreased perfusion in the setting of rapid growth, leading to ischemia, necrosis
(red degeneration) and release of prostaglandins. This hypothesis is supported by the observation that fibroid pain
typically presents in the late first or early second trimester, which corresponds to the period of greatest fibroid
growth. Another theory is that vessels supplying the fibroid become partially obstructed as the uterus grows and
changes its orientation to the fibroid
•Large (at least 3 cm) submucosal and retroplacental fibroids that distort the uterine cavity may be associated with
adverse pregnancy events, including pain, vaginal bleeding, placental abruption, fetal growth restriction, and
preterm labor and birth. (See 'Complications' above.)
•We suggest symptomatic treatment(pain management) of pregnancy complications related to fibroids ( Grade
2C ). Every effort should be made to avoid surgical removal of fibroids in pregnancy because of the risk of
significant morbidity (hemorrhage).
•We suggest elective cesarean delivery prior to the onset of labor if the uterine cavity was entered during a prior
myomectomy or a large number of myomas were removed (eg, greater than 6) ( Grade 2C ). In the absence of
these criteria, we suggest patients with a prior myomectomy be managed similar to women who have had a prior
cesarean delivery .

77- Scenario of ectopic after 6 weeks of amenorrhea home pregnancy +ve whats next
A-Abdominal us
B-Transvaginal us
C-Bhcg
D-Laproscop
DxT amenorrhoea (65–80%) + lower abdominal pain (95 + %) + abnormal vaginal bleeding (65–85%) ectopic
pregnancy
78-A pregnant lady came with severe frontal headache 13 week. She had bp 85/50.pulse 115/min. What
would be your next step of Inv?
a) LP
b) MRI
c) FBE
d) Blood Culture
e) Usg abdmen

DDX 》》SAH, first patients stability, second CT, OR MRI ( best CT )

79- 7 weeks amenorrhoea comes with the blood per vaginum, home pregnancy test was positive,
obesity is obscuring to do the pelvic exam, what to do next?
A-vaginal usg
B-Pelvic usg
C-Serum quantitative bhcg
80-4th Post partum day comes with a fever but no othersymptom, small tear no suture applied?
A-Wound infection
B-Breast engorgemen ( 2-5 th day)
C-Endometritis
D-Endometriosis

PUERPERAL FEVER
JM P 1205
Puerperal fever is defined as raised temperature of ≥  38 ° C from day 1 to day 10. If fever, think of the three
Bs—birth canal, breast, bladder. The cause is genital infection in about 75% of patients. Endometritis presents with
offensive lochia, abdominal pain and a tender uterus. Other causes include urinary tract infection, mastitis and an
intercurrent respiratory infection. Investigations include a vaginal swab for smear, culture and sensitivities (include
anaerobic culture) and a midstream specimen of urine for microscopy and culture, blood culture and an FBE.
Treatment
amoxycillin/potassium clavulanate plus metronidazole (while awaiting sensitivities) Beware of severe puerperal
sepsis such as Gram-negative septicaemia or Clostridium welchii septicaemia and the rare Bacteroides fragilis.

81-39 or 40 weeks pregnant lady in labour. Cervix 5 cm effaced. Membranes ruptured and baby 1 cm
above ischial spine....contractions every 5 minutes. You examined her after 4 hours you find the same
findings(I am sure about that). What is next.?
A) cesarian section
B) oxytocin

NORMAL VAGINAL DELIVERY (NVD)

First stage: begins with regular uterin contractions and ends with complete cervical dilation( 10 cm )
A) Latent phase: begins with mild irregular contractions up to 4 cm cervical dilatation
B) Begins from 4 cm cervical dilatation up to full dilatation ( 10 cm )
• FHR assessment every 15 mins
• active phase must progress at least 1 cm of cervical dilatation per hour
Second stage: delivery of fetus (FHR assessment every 5 mins)
prolonged >3h nulliparous
>2h multiparous  if prolonged operative vaginal delivery(forceps/vaccum)
if obstructed(no progress in fetal station in 1h) c/s
Third stage: delivery of placenta and fetal membrains
After 5-10 mins up to 30 mins normal
Prolonged >30 mins
Active management: prophylactic administration of oxytocin

Fetal station: palpitation of the presenting part of fetus allows the examiner to stablish its station by quantifying
the distance of the presenting part to the maternal ischial spine(-5 to +5).where 0 station is in line with the plane
of maternal ischial spine.
Braxton_Hicks contractions: irregular and no more than twice per hour
Real contractions: starts with with a frequency of once every 15 to 30 mins and increase to every 2-3 mins

82-21weeks pregnant,presents with left iliac pain,vomiying 2times.no rigidity or rebound tenderness.o/e
an anxious lady with mild lower abdominal pain.mild proteinuria,no nitrites in urine?
Uti
Torsion of corpus luteum of pregnancy
Ectopic pg
Round lig tenderness (no nausea or vomiting)

‫ دردش ظاهرا‬sharp ‫ هست و چند ثانیه طول می کشه یعنی‬intermittent ‫ بیشتر سمت راست و گاهی دوطرفه‬،‫هست‬.

The pathogenesis of RLP is varied. Although very common during pregnancy, non-gestating women can
also experience RLP. The most common causes of RLP are as follows:

ROUND LIGAMENT PAIN


RLP may be caused by a spasm or cramp when the ligament contracts involuntarily. The ligament pulls on nerve
fibers and sensitive structures of the female reproductive system. Since the uterus tends to be oriented towards
the right side of the body, the pain is also often felt on the right side. This leads to frequent confusion with
appendicitis. During pregnancy, the uterus expands to accommodate the growing fetus. This increase in size and
weight of the uterus puts stress on the ligament that holds it, causing it to stretch. During physical exertion or
sudden movements, the ligament is overly stretched, causing pain.
The most common symptoms of RLP are:
Sudden pain in the lower abdomen, usually in the right side of the pelvic area that can extend to the groin.
Shooting abdominal pain when performing sudden movements or physical exercise. Pain is sudden, intermittent
and lasts only for a few seconds.
Round ligament pain is a sharp pain or jabbing feeling often felt in the lower belly or groin area on one or both
sides. It is one of the most common complaints during pregnancy and is considered a normal part of pregnancy. It
is most often felt during the second trimester.
Abdominal pains during pregnancy may be due to various pathologies. RLP is one of the most common and benign
of these pains. However, diagnosis of RLP is problematic. Some of the conditions that may present symptoms
similar to those of RLP are appendicitis, ectopic pregnancy, kidney stones, urinary tract infection, uterine
contractions, inguinal hernia, ovarian cysts, and endometriosis. If abdominal pain is continuous and accompanied
by vaginal bleeding, excessive vaginal discharge, fever, chills, or vomiting, then it is most unlikely to be RLP and
immediate consultation with a health care provider is warranted .
Physical examination, ultrasonography, and blood and urine tests may be able to pinpoint the actual cause of
abdominal pain. In some cases, however, RLP was only diagnosed during exploratory surgery.
83-23 year old girl comes with 8 week home pregnancy test positive . On examination, uterus palpable 3
cm from pubic symphisis. What is the cause?
Molar pregnancy
Polyhydraminias
Multiple gestation
Last period in the early pregnancy
Multiple fibroids
‫حاملگی موالر با وازینال بلیدینگ یا عالیمی از هایپرامزیس همراهه‬
‫نه پلی هیدرامنیوس مطرحه‬،‫هفته نه چندقلویی اونقدر رحم رو بزرگ میکنه‬8 ‫تو‬

The most common cause of inappropriate FH / GA is incorrect lmp calculation

84-A pregnant lady comes after taking care of her friends son who was later diagnosed as having EBV
infection. She’s worried. What is your management?
-Ultrasound for hydrops
-Check serology for antibody titres
-Reassure
-Tell her to come if any symptoms arise

EBV infection during pregnancy — There is little evidence of a teratogenic risk to the fetus in women
who develop infection during pregnancy. Transplacental transmission of EBV appears to be rare

JMp270: There are three forms: the febrile, the anginose (with sore throat, see FIG. 28.1 ) and the glandular (with
lymphadenopathy). It may occur at any age but usually between 10 and 35 years; it is commonest in 15–25 years
age group.
Diagnosis
The following laboratory tests confirm the diagnosis of EBM:
• WCC shows absolute lymphocytosis.
• Blood film shows atypical lymphocytes.
• Diagnosis confirmed (if necessary) by EBV-specific antibodies, viral capsule antigen (VCA) antibodies—IgM,
IgG and EB nuclear antigen (EBN-A).
Treatment
• NSAIDs or paracetamol to relieve discomfort
• Gargle soluble aspirin or 30% glucose to soothe the throat
• Advise against alcohol, fatty foods, continued activity, especially contact sports (risk of splenic rupture)
• Ensure adequate hydration
• Corticosteroids reserved for: neurological involvement, thrombocytopenia, threatened airway obstruction. Not
recommended for uncomplicated cases

85-35 y old with lt scrotal swelling,he is not having any problem with that. on ex tests feel normal with
swelling in upper pole of testes.spermatic cord can be felt seperately,next Mx?
a) tumor marker
b)needle aspiration
c)reassure and review after 3 months
d)trans inguinal excision

-١٨ ‫ طبق گفته آكسفورد‬،‫آكسفورد ميگه كه اگر نسبت به تشخيص توده داخل اسكروتوم اطمينان ندارين بررسى هاى بيشتر بايد انجام بشه‬
‫ آپتوديت هم پيشنهاد ميكنه تمام اين وارد بيشتر‬.‫ بعداً مشخص ميشه كه كنسر بودن‬،‫ مواردى كه در ابتدا اپيديديميت تشخيص داده ميشن‬٪٣٣
‫ بنابراين براى اين مورد حتما ً بايد سونو انجام بشه ولى اينجا سونو نداريم پس من با تومور‬.‫بررسى بشن و براى همشون سونو انجام بشه‬
‫ البته اگه تو سوال گفته باشه كه توده سيستيك و بدون ارتباط با‬.‫ ماركر راحت تر هستم‬testis ‫ هست اون موقع هم ميشه‬reassure ‫كرد و‬
‫ هم ميشه‬aspiration ‫ يعنى دو تا گزينه درست ميشه‬،‫انجام داد‬.

Testis mass/cyst  confirmed in us  marker & excision if applicable(no aspiration cause of seeding)
Epididym cyctconfirmed on us  could be observed

86-Melanoma skin care screening. 4o years old IT guys come for screening. What would be the risk for
him? That’s all given in exam. Nothing mentioned.
a. recent diagnosis of melanoma in cousin
b. outdoor working in early twenties
c. history of sunburn in childhood
d. family history of BCC
87-50 y old man is known alcoholic 24 hrs post operative.he is confused what will you check :
1) ABG
2) serum alcohol levels
3)CXR

HB : postop. confusion and disorientation = acute delirium


Most Common = hypoxia,due to:atelectasis,analgesics,anesthetic from surgery,
Mx / Dx – pulse oximetry

Uptodate: The most common PACU complication is postoperative nausea or vomiting (PONV)Respiratory problems
are the second most common complication arising in the PACU.The most common central nervous system
complication in the PACU is an altered state of consciousness (delirium, agitation, somnolence).

Conclusion: as indicated most common cause are hypoxia but it should be suggested in the question by some sx
like tachypnea distress or … but pulsoxymetry is a non invasive easy test for evaluating hypoxia, I think if there is
no strong indication for a specific condition Pulsoxymetry could be the first measure the next one checking urea
and electrolytes , and others as illustrated in mcq 3.045 question.
alcohol withdrawal syndromes
Syndrome Clinical findings Onset after last drink
Minor withdrawal Tremulousness, mild anxiety, 6 to 36 hours
headache, diaphoresis, palpitations,
anorexia, GI upset; Normal mental
status
Seizures Single or brief flurry of generalized, 6 to 48 hours
tonic-clonic seizures, short post-ictal
period; Status epilepticus rare
Alcoholic hallucinosis Visual, auditory, and/or tactile 12 to 48 hours
hallucinations with intact
orientation(sensorium rmains
normal, not delirious) and normal
vital signs
Delirium tremens Delirium, agitation, tachycardia, 48 to 96 hours
hypertension, fever, diaphoresis
88-man 85 yrs old resident of nursing home has mmse of 12/30,he cudnt eat,has urinary n feacal
incontinence…wt is the most appropriate management for poor apetite
a.arrange a family meeting n discuss
b.nasogastric tube
c.TPN
d.dab his lips with wet cotton

The use of nasogastric or nasoenteric tubes in patients with dementia is not recommended as they may pull out
the tube, leading to discomfort for patient and family both, since the patient will have to be reintubated.
Gastrostomy is normally indicated as the route for alternative feeding, since it is already commonplace and laid
down in rules when the duration of ENT is over six weeks [60].The use of Enteral Nutrition Therapy (ENT) or
Artificial Nutrition and Hydration is only indicated when there is a risk of malnutrition and severe impairment of
the swallowing process, with the possible consequence of aspiration pneumonia

When patients with advanced dementia develop a loss of appetite, weight loss, difficulty swallowing or aspiration, 
a discussion of feeding issues should occur without delay. This should involve a multidisciplinary assessment of revi
rsible causes of not eating and discussions with family about the plan of care in relation to the stage of dementia. 

89-WoF mc associated with undescended unilateral testis?


A-Testicular cancer
B-Inguinal hernia
C-Testicular torsion
D-Infertility

Jm p 1064 : The problem of non-descent


• Testicular dysplasia
• Susceptible to direct violence (if in inguinal region)
• Risk of malignant change (seminoma) is 5–10 times greater than normal

Long-term complications
• Relative risk of cancer is 40-fold higher in the undescended testis. Most are seminomas; carcinoma in situ
represents a small percentage (~2%). There is a slightly increased risk of cancer in the contralateral, normally
descended testis.
• Reduced fertility
• Increased risk of testicular torsion
• Increased risk of direct inguinal hernias (due to a patent processusvaginalis)

Inguinal hernia — Almost 90 percent of undescended testes have an associated patent processus vaginalis
Testicular torsion — The incidence of testicular torsion is approximately 10 times higher in undescended testes
than in normal scrotal testes
If only one testis is undescended, the sperm count will be subnormal in 25 to 33 percent and the serum FSH
concentration will be slightly elevated
Testicular cancer — Men with a history of undescended testis have an increased incidence of developing testicular
germ cell cancers of 1 in 1000 to 1 in 2500, versus 1 in 100,000 in the general population. Approximately 10
percent of testicular tumors occur in patients with a history of undescended testicle

90-Old lady presents to the Emergency Department with abdominal distention and pain. She has no
complains of vomiting or diarrhea. She has history of appendectomy which was 20 years ago. Which of
the following is the most likely cause of presentations?
A) Adhesions
B) Colon Cancer
C) Fecal Impaction
D) Volvulus(more likely)

how to differentiate these all four condition by the clinical presentation??

91-head of your ward smells alcohol when he gives round. U told register and he asked not to interfere.
what will you do?
a) inform medical board
b)confront the head
c)inform another senior colleague
inform mirror but one up

92-A 5th day postpartum patient develops vaginal bleeding but afebrile. Patient is allergic to penicillin.
No abnormality detected. Which antibiotics will you give?
A.ceftri+genta
B.Genta+cephotaxime
C.Clinda+metro ??? (clinda+genta better)

SECONDARY POSTPARTUM HAEMORRHAGE


(JM p1205): Secondary postpartum haemorrhage is any bright bleeding from the birth canal 24 hours or more after
delivery and may occur at any time up to 6 weeks postpartum.
It tends to peak at 5–10 days.
Causes
• Retained products of conception (PoC)
• Infection, especially at placental site
• Laceration of any part of the birth canal
• Coagulation disorder
No cause is found in one-third of cases (i.e. idiopathic subinvolution). Investigations: ultrasound, cervical swab for
smear and culture, FBE, IV oxytocin 10 IU, Ergometrine 250 mcg IM, Antibiotics (e.g. amoxycillin/clavulanate +
metronidazole  +  gentamicin while awaiting culture)

94-A bed ridden patient with constipation, from 3 month, now has abdominal pain with distension, no
vomiting, abdomen in exam is tympan (no x-ray), (not mentioned about surgery history) DX?
a. Sigmoid volvulus
b. colon cancer
c. Faecal impaction
d. Adhesion
e. Pseuduobstraction
bed ridden + tympanic percission= volvolus
Patients with volvulus are commonly elderly, debilitated, and bedridden. Often, the patient has a history of
dementia or neuropsychiatric impairment. As a result, only a limited history is available.

More than 60-70% of patients present with acute symptoms; the remainder present with subacute or chronic
symptoms. A history of chronic constipation is common. The patient may describe previous episodes of abdominal
pain, distention, and obstipation, which suggest repeated subclinical episodes of volvulus. Tympan in exam.
Abdominal distention is commonly massive and characteristically tympanitic over the gas-filled, thin-walled colon
loop. Overlying or rebound tenderness raises the concern of peritonitis due to ischemic or perforated bowel. The
patient may have a history of episodes of acute volvulus that spontaneously resolved; in such circumstances,
marked abdominal distention with minimal tenderness may occur.

95- Another similar scenario, DRE is normal, with abdominal x-ray not mentioned about surgery history
(for me like a coffee bean), what is the diagnosis?
a. Sigmoid volvulus
b. colon cancer
c. Faecal impaction
d. Adhesion
e. Pseuduobstraction

95.1- A patient who had a history of sigmoid colectomy for a previous episode of sigmoid
volvulous, now presents with severe abdominal pain and distension, the X ray Abdomen was
given and the typical coffee bean appearance was seen. It was volvulus! What is the most
appropriate management?
1. Hemicolectomy
2. Enema
3. Colonoscopy

Cecal volvulus: A plain, upright abdominal film reveals the classic “comma” or “coffee bean” shaped cecum with an
air-fluid level in approximately 25 percent of patients (image 5 and image 6) [6,29]. The dilated cecum is typically
displaced medially and superiorly, although it can be displaced anywhere in the abdomen [7]. In addition, the
proximal small bowel is distended with air fluid levels while the distal colon is decompressed [9,11]. Cecal bascule
can demonstrate similar findings with a more central position of the dilated cecum (image 7). Free air can be
identified under the diaphragm in the clinical setting of a bowel perforation.

96-Patient on sertraline and selegiline,then come with abdominal pain and diarrhoea and confusion.
1.reduce sertraline
2.reduce selegiline
3.stop sertraline
4.stop selegiline
5.stop both
Serotonin syndrome
97-What is the best treatment for recurrent perirectal fistulas in patients with Crohn's?
1,infliximab
2,Surgery
3,Steroid
4,Mesalamine

— Surgery for anal fistula disease should be considered in patients who have simple low intersphincteric fistulas or
fistulas refractory to medical therapy and those who have severe or disabling symptoms. However, surgery should
not be performed in patients with active proctitis as postoperative diarrhea can be irritating to the wound and may
be associated with prolonged healing.
Patients with severe or refractory disease — Immunomodulator therapy should be considered in patients with
symptomatic perianal fistulas who do not respond to antibiotics and local therapy.

So first AB+local streoids  immunomodulatory therapy  surgery

98- A 6 yrs. old boy came with haematuria and oedema BP: 130/60, U/A: pro +++ , RBC: ++ , no history
of resent URTI, urea and creatinine raised, what is the next management?
a. Bed rest at home and salt restriction
b. Penicillin
c. Prednisolone
d. Admit in hospital for fluid and salt restriction

GLOMERULONEPHRITIS
JM P:868
Glomerulonephritis means kidney inflammation involving the glomeruli. It can be simply classified
into:
• nephritic syndrome: oedema + hypertension + haematuria
• nephrotic syndrome: oedema + hypoalbuminaemia + proteinuria
• asymptomatic kidney disease
NEPHRITIC SYNDROME
The main causes of glomerulonephritis–nephritic syndrome are:
• IgA nephropathy (commonest)
• thin glomerular basement membrane disease (has an AD genetic link)
• post-streptococcal glomerulonephritis
• systemic vasculitis

Tx:
IgA nephropathy: Fluid restriction, Low protein, high carbohydrate, low salt diet
PSGN: general measures include sodium and water restriction, and diuretic therapy

NEPHROTIC SYNDROME
JM P871: Treatment
DxT proteinuria + generalised oedema +  hypoalbuminaemia  nephrotic syndrome
Clinical features
• Proteinuria >3 g/day (3–4 1 on dipstick)
• Swelling of eyelids and face
• Generalised oedema, especially peripheral oedema
• Hypoalbuminaemia <30 g/L
• Hypercholesterolaemia >4.5 mmol/L
• Waxy pallor
• Normal BP
• Dyspnoea
• Frothy urine
Predisposes to sepsis (e.g. peritonitis, pyelonephritis, thromboembolism).
Causes
• 1 in 3 (approx.):— systemic kidney disease (e.g. diabetes, SLE, amyloid)
• 2 in 3 (approx.):— idiopathic nephrotic syndrome (based on kidney biopsy)
— minimal change disease (commonest)
— focal glomerular sclerosis
— membranous nephropathy
— membranoproliferative glomerulonephritis
• Others: drugs, malignancy, infection e.g. malaria
Treatment
• Referral to renal physician or unit
• Bed rest
• Diet: low fluid, high protein, low salt
• Diuretics
• Prednisolone
• Phenoxymethylpenicillin
• Aspirin

99-62 y/o lady menopause from 55 yrs, sexually active purulent vaginal discharge
A-Chlamydia or other non-STI cervicitis
A-Endometrial ca
C-Cervical ca

100_ An old women presents with the history of purulent brownish green vaginal discharge.She only has
sexual relationship with his husband.P/S examination confirms the discharge.Her pap smear previous
were normal.What is the most likely cause of her discharge?
a.chlamydia trachomatis (if no sexualy active or have only with one partner)
b.Endometrial Ca(possible)
c.Cervial Ca(more possible due to discharge)

you know these kind of questions really depend on having a through examination. color, odor and
quality of the discharge( representing infection or not) time of discharge(postcoital, …) onset of
start(more than 4 year from menopause=greater risk of cancer).

You might also like